UK*TE 2022 Flashcards
You see a three-year-old in clinic. Their legs appear as in the image below.
Bilateral varus deformity at the knee.
Which of the following is the most likely diagnosis?
A. Blount’s disease
B. Fibular hemimelia
C. Femoral anteversion
D. Physiological genu varum
E. Renal osteodystrophy
Answer: A. Blount’s disease
- Best divided into two distinct disease entities
o Infantile Blount’s: pathologic genu varum in children 2 to 5 years of age, male > female, more common, bilateral in 50%, centred at the tibia. Diagnosis is suspected clinically with presence of a genu varum/flexion/internal rotation deformity and confirmed radiographically with an increased metaphyseal-diaphyseal angle. RFs: early walking, large stature, obesity. Treatment ranges from bracing to surgery depending on patient age, severity of deformity, and presence of a physeal bar. Langenskiold classification. Bone involvement: proximal medial tibial physis producing genu varus, flexion, internal rotation AND MAY HAVE COMPENSATORY distal femoral VALGUS. Often self limited/bracing/surgery.
o Adolescent Blount’s: pathologic genu varum in children > 10 years of age, less common, less severe, more likely to be unilateral. RFs: Obesity. No radiographic classification. Bone involvement: Proximal tibia physis, AND may have distal femoral VARUS and distal tibia VALGUS. Progressive and never resolves spontaneously – therefore bracing unlikely to work and surgery only option.
- Pathophysiology: likely multifactorial but related to mechanical overload in genetically susceptible individuals including excessive medial pressure produces an osteochondrosis of the medial proximal tibial physis and epiphysis. Osteochondrosis can progress to a physeal bar
Physiologic genu varum:
- Normal in children <2 years
- Genu varum migrates to neutral at ~14 months
- Continues to a peak genu valgum at ~3 years of age
- Genu valgum then migrates back to normal physiologic valgus at ~7 years
Langenskiold Classification
o type I thru IV consist of increasing medial metaphyseal beaking and sloping
o type V and VI have an epiphyseal-metaphyseal bony bridge (congenital bar across physis)
o provides prognostic guidelines: Stage II and IV can exhibit spontaneous resolution.
Presentation:
- Genu varum/flexion/internal rotation deformity – usually bilateral
- Positive ‘cover up’ test.
- Leg length discrepancy
- Usually NO tenderness, restriction of motion, effusion
- Lateral thrust on walking
Radiographic findings suggestive of Blounts disease:
- varus focused at proximal tibia
- severe deformity
- asymmetric bowing
- medial and posterior sloping of proximal tibial epiphysis
- progressing deformity
- sharp angular deformity
- lateral thrust during gait
- metaphyseal beaking: different than physiologic bowing which shows a symmetric flaring of the tibia and femur
Measurements
- Metaphyseal-diaphyseal angle (Drennan): Angle between line connecting metaphyseal beaks and a line perpendicular to the longitudinal axis of the tibia
o >16 ° is considered abnormal and has a 95% chance of progression
o Drennan angles between 11-16° necessitate close observation for the progression of tibia vara
o <10 ° has a 95% chance of natural resolution of the bowing
- tibiofemoral angle: angle between the longitudinal axis of the femur and tibia
Causes of Pathologic Genu Varum:
* persistent physiological varus
* rickets
* osteogenesis imperfecta
* MED
* SED
* metaphyseal dysostosis (Schmidt, Jansen)
* focal fibrocartilaginous defect
* thrombocytopenia absent radius
* proximal tibia physeal injury (radiation, infection, trauma)
A child presents to the Emergency Department having sustained a femoral fracture.
Which scenario would prompt subsequent imaging in the form of a Skeletal Survey?
A. A 2 year old with a femoral fracture sustained playing with older sibling on a trampoline
B. A 6 year old with Cerebral Palsy with a femoral fracture sustained whilst having physiotherapy
C. A 14 year old with a femoral fracture sustained when knocked off their bicycle by a car
D. A 4 month old with a femoral fracture sustained crawling off the end of the bed
E. An 18 month old with a femoral fracture sustained slipping on the wet kitchen floor
Answer: D. A 4 month old with a femoral fracture sustained crawling off the end of the bed
- Pediatric Abuse is the second most common cause of death in children and 50% of fractures in children younger than 1 year of age are attributable to abuse.
- Diagnosis can be suspected with a pediatric injury that is inconsistent with the mechanism of injury, a delay in seeking care, long bone fractures in nonambulatory child, or presence of highly specific fractures.
- Treatment involves reporting abuse to the appropriate agency and hospital admission for multidisciplinary evaluation. Rarely, operative management of fractures may be required.
High specificity fractures
o long bone fractures in non-ambulatory child
o classic metaphyseal lesion
fracture at junction of metaphysis and physis (primary spongiosa)
torsional / traction-shearing strain when infant’s extremity is pulled or twisted violently
high specificity for child abuse
corner fractures: discrete avulsion of the metaphysis
bucket handle fractures: horizontal avulsion fracture with appearance of central and peripheral components gives the appearance of a bucket handle. Avulsed bone fragment may be seen en face
o transphyseal separation of the distal humerus
o rib fractures, especially posteromedial
o scapula fractures
o sternal fractures
o spinous process fractures
* moderate specificity fractures
o multiple fractures in various stages of healing
o vertebral body fractures and subluxations
o digital fractures
o complex skull fractures
* other injuries concerning for abuse - multiple bruises, burns
Skin lesions are most common presentation.
3.
A 12-year-old girl presents to the paediatric clinic with claw toes and a high medial longitudinal arch to both feet. She also has callouses underneath her 5th metatarsal heads.
When examining her feet, which movements around the foot and ankle are likely to be weak
A. Ankle Plantarflexion and Foot Inversion
B. Ankle Plantarflexion and Foot Eversion
C. Ankle Dorsiflextion and Foot Eversion
D. Ankle Dorsiflextion and 1st Ray Plantarflexion
E. Ankle Plantarflexion and 1st Ray Plantarflextion
Answer: C. Ankle Dorsiflextion and Foot Eversion
- Charcot-Marie-Tooth Disease, also known as peroneal muscular atrophy, is a common autosomal dominant hereditary motor sensory neuropathy, caused by abnormal peripheral myelin protein, that presents with muscles weakness and sensory changes which can lead to cavovarus feet, scoliosis, and claw foot deformities.
- Diagnosis is made with nerve conduction studies showing low nerve conduction velocities with prolonged distal latencies in the peroneal, ulnar, and median nerves.
- Treatment involves a multidisciplinary approach to address neuropathy, cavovarus and claw foot deformities, and scoliosis.
- Pathophysiology
o HMSN Type I
abnormal myelin sheath protein is the basis of this degenerative neuropathy.
results in a combination of motor and sensory disturbances.
o HSMN Type 2
intact myelin sheath with wallerian axonal degeneration that results in mild sensory and motor conduction velocities.
o pathoanatomy
peroneus brevis: peroneal involvement is typically first and most profound, results in muscle imbalance and varus deformity
tibialis anterior: weakness results in dropfoot
intrinsic muscles of hand and foot - check for wasting of 1st dorsal interosseous in hands - Genetics
o autosomal dominant duplication of chromosome 17 (most common): codes for peripheral myelin protein 22 (PMP 22) expressed in Schwann cells (most common) or X-linked connexin 32. But may also be autosomal recessive or X-linked. - Orthopedic manifestations: pes cavovarus, claw toes, hip dysplasia, Scoliosis, hand muscle atrophy and weakness
- Peroneus longus (more normal) overpowering weak tibialis anterior and weak intrinsics and contracted plantar fascia
- Varus caused by tibialis posterior (normal) overpowering weak peroneus brevis
Symptoms
* motor deficits: initial symptoms are distal weakness and atrophy of the distal muscles, instability during gait, clumsiness, frequent ankle sprains, difficulty climbing stairs
* lateral foot pain
* sensory deficits are variable
Physical exam
* Cavovarus foot: (similar to Freidreich’s ataxia) with hammer toes or clawing of toes, usually bilaterally and symmetric. Occurs due to unoposed pull of peroneus longus causing plantarflexion of the first ray and compensatory hindfoot varus. Initially flexible, but progresses to a rigid deformity
* motor weakness
o peroneal weakness: weakest muscles around foot and ankle
o anterior tibialis: weakens next, but typically stronger than the peroneals - can lead to drop foot in swing initially and later to a fixed equinus
o posterior tibialis: stays strong for a prolonged period of time
o weak intrinsics - including weak EDB and EHB
o clawtoes
* hyporeflexia or areflexia
* Coleman block test: test to determine if hindfoot varus deformity is secondary to plantar-flexed first ray vs an independent component.
o If deformity corrects with Coleman block, this suggests a forefoot driven varus deformity.
o If deformity does not correct with Coleman block, this suggests hindfoot driven varus deformity.
o a rigid hindfoot will not correct into neutral
* upper extremity: intrinsic wasting of hands, weak pinch and grasp
* spine: scoliosis may be evident on Adam’s forward bend test
- NCS: low nerve conduction velocities with prolonged distal latencies are noted in peroneal, ulnar, and median nerves. Can also see low amplitude nerve potentials due to axonal loss
- Genetic Testing: key component for diagnosis of CMT
o DNA analysis - PCR analysis used to detect peripheral myelin protein 22 (PMP22) gene mutations
o chromosomal analysis: duplication on chromosome 17 seen in autosomal dominant (most common) form
4.
A 12-month old child becomes acutely unwell with a painful knee following varicella infection.
What organism must you rule out?
A. Group A beta-haemolytic streptococcus
B. Group B beta-haemolytic streptococcus
C. Group C beta-haemolytic streptococcus
D. Group G beta-haemolytic streptococcus
E. Group D beta-haemolytic streptococcus
Answer: A. Group A beta-haemolytic streptococcus
Pediatric Septic Hip Arthritis is an intra-articular infection in children that peaks in the first few years of life.
While diagnosis may be suspected by a combination of history, physical exam, imaging, and laboratory studies, confirmation requires a hip aspiration.
Considered a surgical emergency and requires prompt recognition and urgent surgical I&D followed by IV antibiotics.
Hip joint involved in 35% of all cases of septic arthritis
Knee joint involved in 35% of all cases of septic arthritis
Risk factors: prematurity (relatively immunocompromised), cesarean section, patients treated in the NICU, invasive procedures such as umbilical catheterization, venous catheterization, heel puncture may lead to transient bacteremia
Joints with intra-articular metaphysis include: hip, shoulder, elbow, ankle (not KNEE)
Enzymatic destruction: release of proteolytic enzymes (matrix metalloproteinases) from inflammatory and synovial cells, cartilage, and bacteria which may cause articular surface damage within 8 hours
Increased joint pressure: may cause femoral head osteonecrosis if not relieved promptly
Group B streptococcus – most common in neonates with community acquired infection exposed during trans-vaginal delivery
Staph aureus – G+ve cocci in clusters, most common organism in >2years, most common nosocomial infection in neonates.
Neisseria gonorrhoeae – commonest organism in adolescents, g-ve diplococci, usually preceeding migratory polyarthralgia – multiple joint involvement, small red papules
Group A Beta-haemolytic strep – most common following varicella infection
HACEK organisms – Haemophilus (markedly decreased since vaccine), Actinobacillus, Cardiobacterium, Eikenella, and Kingella
Original Kocher Criteria (modified to include CRP)
WBC > 12,000 cells/µl of serum
inability to bear weight
fever > 101.3° F (38.5° C)
ESR > 40 mm/h
* Probability of septic arthritis may be as high as 99.6% when all four criteria above are present, if none of the above predictors are present, probability of having septic arthritis is <0.2%, 3% incidence of septic arthritis if 1/4 criteria present, 40% incidence if 2/4 criteria present, 93% incidence if 3/4 criteria present
5.
You review a 4-year-old with genu valgum of 14 degrees. Mum is concerned, what is your treatment plan?
A. Hemiepiphysiodesis
B. Bracing KAFO
C. Bracing and restricted weight bearing
D. Observation
E. Medial tibial epiphysiodesis
Answer: D. Observation
• Genu Valgum is a normal physiologic process in children which may also be pathologic if associated with skeletal dysplasia, physeal injury, tumors or rickets.
• Diagnosis is made clinically with presence of progressive genu valgum after the age of 7.
• Treatment is observation for genu valgum <15 degrees in a child <7 years of age. Surgical management is indicated for severe and progressive genu valum in a child > 7 years of age.
Distal femur is the more common location of pathological deformity.
Risk factors: prior infection or trauma, vit D deficiency/rickets, obesity, skeletal dysplasia, lysosomal disease.
- Genu varum <2 years
- Genu varum migrates to neutral at ~14 months
- Continues to a peak genu valgum at ~3 years of age (tibiofemoral angle 15-20 deg)
- Genu valgum then migrates back to normal physiologic valgus at ~7 years
o after age 7 valgus should not be worse than 12 degrees of genu valgum
o after age 7 the intermalleolar distance should be <8 cm
- lateral deviation of mechanical axis: decreased growth from lateral physis relative to medial physis
- patellar instability: increased Q-angle and shallow lateral femoral sulcus (lateral femoral condyle growth suppressed predisposing to lateral subluxation)
Associated conditions
- bilateral genu valgum: physiologic, renal osteodystrophy (renal rickets), skeletal dysplasia (Morquio syndrome, spondyloepiphyseal dysplasia, chondroctodermal dysplasia(Ellis-van Creveld))
- unilateral genu valgum: physeal injury from trauma, infection, or vascular insult, proximal metaphyseal tibia fracture (Cozen Phenomenon), benign tumors (fibrous dysplasia, osteochondromas, enchondromas), fibular hemimelia
• Normal lateral distal femoral angle (LDFA) = 85-90 degrees
• Normal medial proximal tibia angle (MPTA) = 85-90 degrees
• Hypoplastic lateral femoral condyle with shallow lateral femoral sulcus
• mechanical axis
o center of femoral head to center of ankle should pass through center of knee
o lateral deviation of mechanical axis in genu valgum, therefore lateral femoral condyle and lateral tibia plateau are subjected to increased loads
• mechanical loading on physis modulates growth
o Hueter–Volkmann law
compression inhibits growth
distraction stimulates growth
o greater proportion of change in growth rate from hypertrophic zone (75%) than proliferative (25%)
greater effect on growth seen from change in size of chondrocytes than number
Management:
- Non-operative is first line treatment – observation and medical management of risk/pathological factors. Bracing is rarely used (may provide temporary relief, but in-effective as long term solution).
o Indications: children <7 years old, tibiofemoral angle <15 deg
o Vast majority of physiological genu valgum will resolve spontaneously, medical management underlying etiology may slow progression.
- Operative:
o Indications: tibiofemoral angle >15 deg, intramalleolar distance 10cm after 10 years old, rapidly progressive deformity after 7 years.
o Medial hemi-epiphysiodesis (extra-periosteal placement to avoid physeal injury): usually temporary (8 plates), permanent hemi-epiphysiodesis (modified Phemister technique)
o Osteotomy: insufficient growth remaining to correct deformity with hemi-epiphysiodesis, skeletally mature patients, non-functional growth plates
Lateral distal femur opening wedge osteotomy: angular correction can be adjusted to desired correction, but requires grafting and is a less stable construct requiring prolonged immobilisation to allow graft to heal.
Medial distal femur closing wedge osteotomy: stable osteotomy, shorter period immobilisation, avoids distracting CPN, but more technically demanding to remove precise angular wedge.
HTO
- Deformity after a proximal metaphyseal tibia fracture (Cozen) should be observed as most remodel - maximum magnitude of deformity reached approximately 12-18 mo after injury, resolve spontaneously within 2-4 years.
6.
You review a 10-year-old boy with a varus deformity 2 years post fracture dislocation of the ankle.
There is a bony bridge involving 20% of the physis on CT scan.
How will you treat him?
A. Observation
B. Observation until skeletal maturity then realignment osteotomy
C. Bracing in AFO
D. Excision of physeal bar and interposition graft
E. Supramalleolar osteotomy
Answer: D. Excision of physeal bar and interposition graft
Physeal bridges commonly occur after Salter-Harris (SH) physeal fractures in early adolescence when the physis is thickest and the cartilage is weakest. Small (<25% of physeal area), central bridges have a better prognosis. There are 3 types of physeal bridges:
(1) Peripheral bridges cause angular deformity and may be amendable to excision.
(2) Elongated bridges involve involve the middle of the physis and are most commonly caused by SH III or IV fractures.
(3) Central bridges have a perimeter of healthy physis but act as a central tether and can tent/distort the articular surface.
When bar resection is performed, interposition options includes fat, cranioplast, bone wax, cartilage, muscle and silicone.
Growth arrest
• medial malleolus SH IV fractures have the highest rate of growth disturbance
• risk factors
o degree of initial displacement: 15% increased risk of physeal injury for every 1mm of displacement
o residual physeal displacement > 3mm - can represent periosteum entrapped in the fracture site, degree initial displacement greatest risk factor for premature physeal closure
o high-energy injury mechanism
o SH III and IV fractures
• types
o partial arrests can lead to angular deformity
distal fibular arrest results in ankle valgus deformity
medial distal tibia arrest results in varus deformity
o complete arrests can result in leg-length discrepancy
• treatment
o angular deformity
physeal bar resection
if < 20 degrees of angulation with < 50% physeal involvement and > 2 years of growth remaining
osteotomy: angular deformity >10-20degrees is an indication for osteotomy as the deformity will not correct spontaneously after bridge resection.
ipsilateral fibular epiphysiodesis
bar of >50% physeal involvement in a patient with at least 2 years of growth
fibular epiphysiodesis helps prevent varus deformity
o leg-length discrepancy
physeal bar resection
if < 50% physeal involvement and > 2 years of growth remaining
contralateral epiphysiodesis if near skeletal maturity with significant expected leg-length discrepancy
7.
You review a patient with osteochondritis dissecans whose MRI scan shows a detached lesion of 4 cm square area.
How would you treat them?
A. Microfracture
B. Fixation with buried variable pitch screw
C. Autograft OATS
D. Autologous chrondrocyte implantation
E. Non-operative management
Answer: C. Autograft OATS
• Osteochondritis Dissecans is a pathologic lesion affecting articular cartilage and subchondral bone with variable clinical patterns.
• Diagnosis may be made radiographically (notch view) but MRI usually required to determine size and stability of lesion, and to document the degree of cartilage injury.
• Treatment may be nonoperative with restricted weight bearing in children with open physis. Surgical treatment may be indicated in older patients (closed physis), lesions that are unstable and patients who have failed conservative management.
Knee is the most commonly affected, with the posterolateral aspect of medial femoral condyle in 70% of cases.
Wilson’s Test: pain with internally rotating the tibia during extension of the knee between 90 and 30 deg, then relieving the pain with tibial external rotation
MRI: characterises size of lesion, status subchondral bone and cartilage, signal intensity surrounding lesion, presence loose bodies.
Osteochondral grafting: indicated in lesions >3cm
Osteochondritis dissecans (OCD) is a disorder of subchondral bone, which secondarily affects the overlying articular cartilage. Separation of a fragment of articular cartilage, complete with a layer of its subchondral bone, occurs in end-stage disease.1 The layer of bone can be very thin, or even absent. Patients may be asymptomatic or present with pain and mechanical symptoms. The knee is most commonly affected. It classically affects skeletally immature patients who are keen on sport. In this group, the condition is usually described as juvenile OCD, but also occurs in adults for whom the prognosis is worse.2
Although OCD was first named by Konig in 18873 its cause, management and prognosis are still not fully understood. It has been our observation, and also that of others, that the proposed management strategies for patients are highly variable.4
With appropriate treatment, it is possible to heal lesions, particularly in juveniles.5 However, lesions that do not heal can lead to premature joint degeneration.4,5 Therefore, early identification and treatment of OCD is essential in order to maximise healing potential.
OCD is one of several types of osteochondral defect. These can be divided into focal and degenerative lesions. Focal defects are well delineated and include OCD, traumatic chondral and osteochondral fractures and osteonecrosis, whereas degenerative defects are typically poorly demarcated.6 Differentiation between the types of defect is based on a combination of history, clinical and radiological findings, and is essential as it will affect the management, and possibly the outcome.
Presenting symptoms for OCD will vary according to the pathological stage of the lesion and its site. In juvenile OCD, if the lesion is still attached and is stable, the symptoms (if there are any) are of vague poorly localised pain, which is often longstanding, and exacerbated by activity, and can intermittently cause an antalgic gait.11 Quadriceps atrophy may be present if there has been a long history.1 Unfortunately, late diagnosis is common due to the vague nature of the symptoms and lack of awareness of the diagnosis among sports coaches, parents and medical staff.
In more advanced cases, where lesions are unstable or displaced, mechanical symptoms such as locking and catching occur, and may be associated with swelling.
Diagnostic imaging is valuable. Plain radiographs including a flexed anteroposterior ‘tunnel view’ will allow most mature lesions to be identified.4,8 MRI is used to identify all OCD lesions and to determine their stability.12
Within the knee, OCD lesions are most commonly seen on the lateral aspect of the medial femoral condyle. In a large multicentre study involving over 500 knees, Hefti et al5demonstrated that 77% of lesions affect the medial femoral condyle, of which 51% are on the lateral aspect, 19% central and 7% medial. In comparison, only 17% were in the lateral femoral condyle, 7% in the patella and only 0.2% arose from the tibial plateau. Lesions of the trochlea have been reported as being the most rare, occurring in 1% of patients.13 In young elite athletes, it is possible that trochlear lesions are more common than this, and patellar lesions less prevalent. However, this is based on our anecdotal experience, and there is no evidence in the literature to support this.
Classification Systems:
XR – Berndt & Hardy: Stage 1-4
MRI – Dipaola et al: Type I - IV
Arthroscopy – Guhl: Type I - IV
The most important aspect of any classification of OCD is to establish whether the lesion is stable or unstable as this will influence how the lesion is treated.
Prognosis
A review by the Research on OsteoChondritis of the Knee (ROCK) study group reported rates of healing for OCD lesions being between 50% and 98%.4
Patient age is a significant factor, which affects the prognosis. Adult onset symptoms are associated with worse outcomes than those found in skeletally immature patients.5 Furthermore, in skeletally immature patients, a younger age is also associated with a more favourable outcome.16
The prognosis also varies with the size and site of the lesion, and the stability of the OCD fragment.5 Medial femoral condylar lesions, narrow lesions and cysts, when present, of < 1.3 mm when visualised on MRI are good prognostic factors16whereas patellar lesions have a poor prognosis.7 This may be due to these lesions having a higher rate of instability compared with condylar lesions.13
If, at diagnosis, the situation is favourable in terms of patient age, location and size of the lesion, these patients will do significantly better with conservative treatment than with precipitant surgery. However, if there are signs of separation of the lesion operative treatment will produce better results.5
Treatment
There are two main aims of treatment of OCD. One is to promote the healing of subchondral bone and the overlying articular cartilage, and the other is to ensure joint congruity in order to protect the opposing joint surface and mitigate circumstances which may bring about the development of osteoarthritis.
Stable lesions
Stable lesions should usually be treated non-operatively, in view of the potential for healing, particularly in juvenile OCD.5 However, some cause persisting pain and although radiographically ‘stable’, benefit from the operative treatment described later in this paper.
Non-operative treatment
The rationale for treatment is to reduce the loading from the affected articular cartilage to facilitate spontaneous healing of the defect, thus preserving the patient’s native hyaline cartilage. In the literature, several authors make recommendations for the conservative treatment of OCD including Kocher et al1and Wall et al,17 who suggest an initial period of six weeks immobilisation in a cast with protected weight-bearing. However, their progression of treatment after this period varies. Kocher et al1 advocate removal of immobilisation and increased weight-bearing over the next six weeks, at which point if the patient is free of pain, a gradual return to sport starting at three to four months. However, Wall et al17 would continue with the initial immobilisation for a further six weeks unless radiographs demonstrated re-ossification of the lesion. Other non-operative treatments have included modification of activities, bracing and physiotherapy, but there is no evidence available regarding specific protocols. Although the evidence for specific treatments is inconclusive, there is agreement that conservative treatment should continue for at least three months.4
In any event, if symptoms are not improving, surgery is indicated and the MRI helps to guide this. If pain has resolved and the MRI is improved, weight-bearing exercises are begun and a graded increase in activity is started to enable return to sports after a further four weeks. If the MRI is not resolving but pain is better, running or sport is still not allowed for another 12 weeks, after which the MRI is repeated. If there is still no improvement, then operative treatment is undertaken. However, if the MRI is improved then activity is increased as already described.
Symptomatic OCD after failed conservative treatment
Although there is a consensus that surgery should be offered when patients have not responded to conservative treatment, there is no clear evidence to show which surgical procedure should be used, or at what time.18 A guiding principle is that the best articular cartilage is the patient’s own, and so the treatment strategy is to preserve the articular surface.
Symptomatic radiographically stable lesions
The MRI will show bone oedema adjacent to the lesion but no separation of the lesion. Although there is a consensus that surgical treatment is indicated in these cases there is no clear evidence which technique is most effective. However, arthroscopic drilling of these lesions is a common treatment which has been shown to be successful, particularly in juveniles.1,8 Both retrograde and transarticular drilling techniques have been described1 and can lead to healing. We recommend arthroscopically-assisted drilling of the lesion directly through the articular surface using a 1.2 mm diameter Kirschner-wire. The drilling should be orthogonal to the joint surface and so supplementary portals may be needed, including if necessary, drilling via the patellar tendon. If there is any doubt about stability, bio-absorbable pin fixation is used) in addition to drilling of the lesion. Typically, two to four of these implants are required and the lengths are chosen to avoid violating the growth plate. Though drilling through healthy articular cartilage may be criticised, it is effective and the joint surface damage is minimal. Nevertheless, some surgeons undertake retrograde drilling with fluoroscopic guidance or the use of drill guides. This is technically more difficult and it is harder to achieve accurate placement and depth,8 increasing the risk of surgical morbidity. It is often hard to locate these lesions visually and probing can be essential to reveal a softer area corresponding with the position of the lesion shown on MRI.
Unstable lesions
Unstable lesions are treated surgically, except for those that are asymptomatic (often in adults) identified by chance on imaging and that have clearly remained in situ for a long time. These require follow-up with serial imaging.
To safeguard joint congruity, the aim is to prevent dislodgment of undisplaced OCD fragments or to replace and maintain displaced fragments within their defect area. Every effort should be made to salvage displaced fragments, as the native hyaline cartilage of the fragment is thought to be superior to the hybrid hyaline-like cartilage provided by techniques, such as microfracture and autologous chondrocyte implantation (ACI).19 Even a loose body, which detached months beforehand, will have living articular cartilage at its surface, which has been nourished via the synovial fluid and therefore continued to grow. Whether a lesion has detached from its host completely, or the lesion is still attached peripherally to healthy articular cartilage, the displaced fragment will have grown to be too big for its host crater. Although traditionally separated lesions were removed, there is still potential for healing, even when surgical treatment is delayed. Even apparently unfavourable lesions can be salvaged, leading us to believe that the outcomes are more positive than previously thought, but the operative technique needs to be scrupulous. Simply pushing these lesions back and ‘fixing in situ’ will not work as the adherent fibrous tissue must be removed first20 to ensure a close fit and stimulate healing.
Multiple techniques for OCD fixation have been described9,18,20-23 and include metallic and bio-absorbable implants and biological fixation using osteochondral plugs. Although there is a consensus that symptomatic patients should be offered surgery, there is no reliable evidence or consensus as to which specific surgical technique should be used.
An arthroscopy is first carried out to retrieve any loose bodies and to assess whether arthroscopic surgery is possible. Although the procedure is commonly carried out arthroscopically,8 we recommend open surgery in most cases as greater access allows the steps below to be carried out more easily and thoroughly. Although there is increased surgical morbidity, the chance for better preparation of the displacement and fixation often offers greater healing potential. The base of the lesion is curetted clear of fibrous tissue and the shiny, corticated surface is prepared with a burr to increase bleeding and promote healing. It is also drilled to create ‘vascular channels’ in order to maximise the chance of revascularisation.
The fragment is then trimmed using a scalpel in order to fit it into the host area. Rather than widening the crater, we recommend trimming the fragment in the belief it is easier for a smaller fragment to revascularise. With a wet gauze swab to hold the fragment more safely, its bony surface can be ‘freshened’ with bone nibblers or a burr used with great care.
Once the fragment has been resized, a decision regarding the type of fixation should be made. There are multiple implants available, all with advantages and disadvantages, and therefore the choice of fixation should be based on the size and type of fragment to be fixed.
Standard metal small fragment cancellous lag screws provide good compression,9whereas the threads of some other metal screws are too fine to grip properly. Metal screws have the disadvantage of making MRI interpretation difficult and also that their heads need countersinking with resultant articular cartilage damage. They should also be removed after one to two years as they have a tendency to back out even with good healing of the lesion, and can damage the articular cartilage of the opposing joint surface.4 If the fragment has a reasonable layer of bone, as is commonly the case, our fixation of choice is a small metallic cancellous lag screw as we feel this offers the best compression.
Bioabsorbable screws are at risk of breaking during tightening in hard, corticated bone even with pre-tapping and it is possible that they fail to provide adequate compression, for a sufficient duration, for healing to occur.23 However, if the fragment is almost entirely articular cartilage, absorbable devices such as barbed nails are more appropriate. In fragments that are too fragile for any fixation device, peripheral suturing can suffice. With this technique there is a risk of sutures ‘cutting out’. This risk may be reduced by lubricating 6/0 vicryl with sterile petroleum jelly taken from dressings.
An alternative fixation is a mosaicplasty of osteochondral plugs placed through the mobile fragment. However, again there is only limited evidence for this method.24
The presence of an underlying subchondral cyst can affect healing and therefore should be addressed. The walls of the cyst should be curetted and drilled to increase vascularisation and to form a favourable cavity into which to impact bone graft. This can be harvested locally from the tibial metaphysis or, if large amounts are needed, the iliac crest. Cysts of up to 2 mm to 3 mm do not require grafting. Bone grafting may also be required in large defects, even without cysts, in order to support the fragment and restore joint congruency.4,8
In adverse lesions, osteotomy or patellar re-alignment procedures may be considered as discussed below.
Unsalvageable lesions
Fixation of OCD lesion is not always possible due to excessive fragmentation or the small size of the fragment, in which case the fragment should be excised. Although the presence of an unfilled defect will lead to osteoarthritis, the results of removal of the fragment can be surprisingly good and sustained, particularly in smaller lesions. Therefore, if the diameter of a lesion in this situation is 2 cm or less, this should be the first choice of treatment.
Should symptoms persist, or the lesion is larger, chondral resurfacing techniques should be considered. Microfracture drilling has been recommended for chondral lesions between 2 cm2 and 4cm2 in size.21 However, the subchondral bone needs to be intact. In contrast to traumatic chondral lesions this is rarely the case in OCD and it should be remembered that treatment with microfracture alone does not restore joint congruency.
Autologous chondrocyte implantation (ACI) or matrix- induced autologous chondrocyte implantation (MACI) has the same issue of not restoring congruity, although the bony defect can be dealt with using bone grafting. Peterson et al25demonstrated favourable outcomes following ACI in 58 patients with OCD of the knee at a mean follow-up of 5.6 years. However, in some studies of ACI and MACI, the type of chondral defect is not clearly specified.26,27 In OCD, due to the pathological subchondral bone, it is possible that the effectiveness of ACI could be impaired compared with its use for chondral defects following trauma.
Osteochondral grafting, whether using autologous grafts from ‘non-weight-bearing’ surfaces of the joint, or allograft, can address the underlying bony defect. Although osteochondral autologous transplantation (OATs) is thought to have better long-term results than microfracture for the treatment of juvenile OCD, particularly in lesions over 2 cm,21 it can have problems with donor-site morbidity. Osteochondral allografts remove the problem of donor site morbidity and have been successful,28 however, there are technical challenges and the long-term results in OCD patients are unclear.1
In addition to the size and location of the lesion, other factors that affect their healing potential should also be taken into account.
Osteotomy should be considered to address adverse limb alignment, thereby offloading the lesion. Applying the same logic, patellofemoral re-alignment procedures may need to be considered. In a small number of older patients, focal metallic implants (e.g., Hemicap, Arthrosurface, Franklin, Massachusetts) may be appropriate.
8.
You review a child in A&E with a proximal femoral fracture. At what age does the greater trochanter ossification centre appear?
A. 3 years
B. 4 years
C. 5 years
D. 6 years
E. 7 years
Answer: B. 4 years
Greater trochanter appears by 4 years, closes at 16-17 years
Lesser trochanter appears by 14 years, closes at 16-17years
Femoral head appears by 1 year, closes by 16-17 years (F), 17-18years (M)
Proximal femur anatomy
• femoral head: center of femoral head should be at the level of the tip of the greater trochanter
o femoral neck: anteverted 15 degrees (in relation to femoral condyles), neck shaft angle of 125 degrees
Blood supply to femoral head:
• Birth to 4 years of age: medial and lateral circumflex and ligamentum teres
• 4 years of age to adult: posterosuperior and posteroinferior retinacular vessels from medial femoral circumflex. Therefore piriformis start nails damage posterosuperior retinacular vessels and can cause AVN of femoral head
• adult age: medial femoral circumflex, therefore avoid transection the quadratus during posterior approach and damaging the MFC artery
• Ligamentum teres: arterial branch of the posterior division of the obturator artery to the femoral head - not significant in adults
• Abdominal aorta
o external iliac artery
common femoral artery: at risk during screw placement in anterosuperior quadrant during THA
profunda femoris
lateral femoral circumflex: ascending branch at risk during the direct anterior approach
medial femoral circumflex: major blood supply to femoral head, at risk during psoas tenotomy
femoral artery perforators: supply vastus lateralis
o internal iliac artery
obturator (posterior branch): supplies transverse acetabular ligament - at risk with screw placement and acetabular retractors in the anteroinferior quadrant
superior gluteal
inferior gluteal: supplies short external rotators and gluteus maximus. Runs along the piriformis after it exits the greater sciatic notch
internal pudendal
re-enters pelvis via lesser sciatic notch
• Corona Mortis
o vascular connection between
inferior epigastric branch of the external iliac vessels
obturator vessels
9.
You perform a rotational profile on a patient in clinic. What is the normal femoral anteversion at birth?
A. 0-10 degrees
B. 10-20 degrees
C. 30-40 degrees
D. 40-50 degrees
E. 50-60 degrees
Answer: C. 30-40 degrees
Is based on degree of anteversion of femoral neck in relation to the femoral condyles
o at birth, normal femoral anteversion is 30-40°
o typically decreases to normal adult range of 15° by skeletal maturity
o minimal changes in femoral anteversion occur after age 8
• Femoral Anteversion is a common congenital condition caused by intrauterine positioning which lead to increased anteversion of the femoral neck relative to the femur with compensatory internal rotation of the femur.
• Diagnosis is made clinically with the presence of intoeing combined with an increase in internal rotation of the hip of greater than 70° with an accompanying decrease in external rotation of the hip of less than 20°.
• Treatment is observation with parental reassurance as most cases resolve by age 10. Rarely, surgical management is indicated in the presence of less than 10° of hip external rotation in children greater than 10 years of age.
• Femoral anteversion is characterized by
o increased anteversion of the femoral neck relative to the femur
o compensatory internal rotation of the femur
o lower extremity intoeing
• There are three main causes of intoeing including
o femoral anteversion (this topic)
o metatarsus adductus (infants)
o internal tibial torsion (toddlers)
• Pathophysiology
o a packaging disorders caused by intra-uterine positioning
o most spontaneously resolve by age 10
• Associated conditions
o can be seen in association with other packaging disorders
DDH
metatarsus adductus
congenital muscular torticollis
10.
On post-natal checks it was discovered that a newborn had partial absence of the posterior ring of C1. Conservative management was employed but he is now 5 years old and is affected by head tilt, suboccipital pain and decreased range of movement about the c‐spine.
What would you offer him?
A. Excision of tumour
B. Immobilisation and NSAIDs
C. Distal and proximal release
D. Posterior fusion
E. Observation
Answer: D. Posterior fusion
Congenital anomalies of the posterior arch of the atlas (C1) are relatively common anomalies. They may range from partial defects presenting as clefts to complete absence of the posterior arch (aplasia).
These anomalies are classified according to Currarino. It should not be confused with Currarino triad (an inherited congenital disorder of the sacrum and anus or rectum).
Currarino Classification: combination of morphology and clinical presentation
* morphological types
o type A: failure of posterior midline fusion of the two hemiarches
o type B: unilateral defect
o type C: bilateral defects
o type D: absence of the posterior arch, with persistent posterior tubercle
o type E: absence of the entire posterior arch, including the tubercle
* clinical subgroups
o 1: incidental imaging finding, asymptomatic
o 2: neck pain or stiffness after trauma to the head or neck
o 3: chronic symptoms referable to the neck
o 4: various chronic neurological problems
o 5: acute neurological symptoms following minor cervical trauma
Type A and subgroup 1 are by far the commonest (approximating 80% of cases) and are encountered in 4% of the general population 7. In contrast, all other morphological types (B to E) are encountered in only 0.69% of the population
This anomaly is a developmental failure of chondrogenesis (lack of chondrification). In the embryological period C1 is usually formed from three primary ossification centres:
* an anterior centre developing into the anterior tubercle
* two lateral centres giving rise to the lateral masses and posterior arch
In ~2% of the population, an additional ossification centre develops in the posterior midline, subsequently forming into a posterior tubercle.
During ossification different anomalies can develop, comprising:
* median cleft(s) of the posterior arch
* varying degrees of posterior arch dysplasia
o either with or without the presence of posterior tubercle (see above)
Fusion of ossicles usually occurs during age 3 to 5 years. Incomplete posterior fusion may even be normal in children up to 10 years old.
Associations:
* Arnold-Chiari malformation
* gonadal dysgenesis
* Klippel-Feil syndrome
* Down syndrome
* Turner syndrome
Treatment and prognosis
Anomalies of the posterior arch of C1 are usually considered benign 4, but may give rise to severe neurological compromise. Especially groups C and D (i.e. isolated posterior ossicle) may be considered a risk factor for neurological morbidity rather than a developmental variant of normal 7.
In cases of doubt (e.g. post-traumatic symptoms not clearly related to the anomaly, type C or D without symptoms) imaging studies may include 7:
* cervical lateral plain radiography, optimally as flexion and extension study
* cervical CT with multiplanar reconstruction and/or 3D
* cervical MRI, possibly with the neck in extension
o including medullary and soft-tissue sequences to depict spinal cord changes and mapping of ligamentous structures
For asymptomatic cases, no treatment or follow-up is needed, as considered a benign anatomical variant. Some authors including Currarino suggest advising patients with type C and D to avoid contact sports. Prevention of cumulative damage to the cord by surgery at an early stage may also be prudent in these types with a posterior tubercle. In cases with symptomatic compression, surgery with excision of the posterior arch is considered curative 7.
Differential diagnosis
When patients with arch anomalies present with trauma, the radiographs may be confusing and misleading. Hence thorough knowledge of these abnormalities is essential to avoid misinterpretation as an osteolytic lesion, fracture or dislocation, e.g. atlantoaxial subluxation.
11.
An 11-year-old girl presents with a left thoracic rib hump. Neurological examination reveals absent abdominal reflexes. X‐rays show a 30-degree curve.
What is the next appropriate step?
A. Observation
B. Bracing
C. MRI
D. Anterior spinal fusion
E. Posterior spinal fusion
Answer: C. MRI
MRI should extend from posterior fossa to conus – purpose is to rule out intraspinal anomalies.
Other indications for MRI:
- Atypical curve pattern (left thoracic, short angular curve, apical kyphosis)
- Rapid progression
- Excessive kyphosis
- Structural abnormalities
- Neurological symptoms or pain
- Midline skin defects (hairy patches, dimples, naevi – signs of spinal dysraphism), café au lait spots,
- Foot deformities (suggest neural axis abnormalities)
- Asymmetric abdominal reflexes (syringomyelia). A syrinx is associated with abnormal abdominal reflexes and a curve without significant rotation.
Adolescent Idiopathic Scoliosis is a coronal plane spinal deformity which most commonly presents in adolescent girls from ages 10 to 18 (10:1 female to male ratio for curves >30deg).
Cobb angle >10deg = scoliosis
Spinal Balance: coronal balance is determined by alignment of C7 plumb line to central sacral vertical line. Sagittal balance is based on C7 plumb from centre of C7 to the posterior-superior corner of S1.
Stable zone = between lines drawn vertically from lumbosacral facet joints.
Stable vertebrae = most proximal vertebrae that is most closely bisected by central sacral vertical line.
Apical vertebrae = vertebra deviated farthest from the centre of the vertebral column.
Diagnosis is made with full-length standing PA and lateral spine radiographs.
Right thoracic curve is commonest, left thoracic curves are rate and indicate MRI to rule out cysts or syrinx.
Treatment can be observation, bracing, or surgical management depending on the skeletal maturity of the patient, magnitude of deformity, and curve progression.
Curve progression risk factors:
- Curve magnitude: >25 deg before skeletal maturity will continue to progress, after skeletal maturity thoracic curves >50deg will progress at 1-2deg/year and lumbar curves >40deg will progress 1-2deg/year.
- Remaining skeletal growth: Peak growth velocity is the best predictor of curve progression, <12 years at presentation, Tanner stage <3, Risser 0-1 (Risser 0 covers the first 2/3rd of the pubertal growth spurt and correlates with the greatest velocity of skeletal growth). Open triradiate cartilage. If curve is >30deg before peak height velocity, there is a strong likelihood of the need for surgery.
- Curve type: thoracic curves more likely to progress than lumbar, and double curves more likely to progress than single curves.
12.
You review an x-ray of a 1-year-old girl with DDH.
What is the name of the horizontal line drawn through the tri-radiate cartilages on a pelvic radiograph?
A. Shenton
B. Perkin
C. Acetabular index
D. Hilgenreiner
E. Klein
Answer: D. Hilgenreiner
Hilgenreiner’s line: horizontal line through the right and left triradiate cartilage – femoral head ossification should be inferior to this line.
Perkin’s line: line perpendicular to Hilgenreiner’s line through a point at the lateral margin of the acetabulum – femoral head ossification should be medial to this line.
Shenton’s line: arc along the inferior border of the femoral neck and the superior margin of the obturator foramen – arc line should be continuous.
Delayed ossification of the femoral head is seen in cases of dislocation.
Acetabular teardrop not typically present prior to hip reduction for chronic dislocations since birth – development of teardrop after reduction is thought to be a good prognostic sign for hip function.
Acetabular Index: angle formed by Hilgenreiner’s line and a line from a point on the lateral triradiate cartilage to a point on lateral margin of acetabulum – should be <25deg in patients older than 6 months.
Centre-edge angle of Wiburg: angle formed by Perkin’s line and a line from the centre of the femoral head to the lateral edge of the acetabulum, <20deg is considered abnormal, only reliable in patients >5years.
Developmental Dysplasia of the Hip is a disorder of abnormal development resulting in dysplasia, subluxation, and possible dislocation of the hip secondary to capsular laxity and mechanical instability.
Diagnosis can be confirmed with ultrasonography in the first 4 months and then with radiographs after femoral head ossification occurs (~ 4-6 months).
Treatment varies from Pavlik bracing to surgical reduction and osteotomies depending on the age of the patient, underlying etiology, and the severity of dysplasia.
Risk factors: firstborn, female, breech, packing disorders, family history, macrosomia, limited hip abduction, talipes, swaddling
Dysplasia: shallow or underdeveloped acetabulum
Subluxation: displacement of the joint with some contact remaining between the articular surfaces
Dislocation: complete displacement of the joint with no contact between the original articular surfaces
Teratologic hip: dislocated in utero and irreducible on neonatal exam, presents with a pseudoacetabulum - associated with neuromuscular conditions and genetic disorders. Commonly seen with arthrogryposis, myelomeningocele, Larsen’s syndrome, Ehlers-Danlos
Late (adolescent) dysplasia: mechanically stable and reduced but dysplastic
Typical deficiency is anterior or anterolateral acetabulum, BUT in spastic cerebral palsy acetabular deficiency is posterosuperior – needs to be considered when planning pelvic osteotomies!
Dysplasia leads to subluxation and gradual dislocation, repetitive subluxation of the femoral head leads to the formation of a ridge of thickened articular cartilage called the limbus.
Chronic dislocation leads to the development of secondary barriers to reduction: pulvinar thickens, ligamentum teres thickens and elongates, TAL hypertrophies, hip capsule and iliopsoas form hourglass configuration. Anatomic changes include increased femoral anteversion, flattening of the femoral head, increased acetabular anteversion, increased obliquitiy and decreased concavity of the acetabular roof, thickening of the medial acetabular wall.
Associated with other packing deformities: congenital muscular torticollis (20%), metatarsus adductus (10%), congenital knee dislocation.
Barlow: dislocates a dislocatable hip by adduction and depression of a flexed femur – ‘click of exit”
Ortolani: reduces a dislocated hip by elevation and abduction of the flexed femur – ‘click of entry’
Galeazzi (Allis): apparent limb length discrepancy due to unilateral dislocated hip with hip flexed at 90 degrees and feet on the table – femur appears shortened on dislocated side.
Klisic test: for bilateral dislocations: line from long finger placed over the greater trochanter and the index finger over the ASIS should point to the umbilicus – if the hip is dislocated, the line will point halfway between the umbilicus and pubis.
13.
A 13-year-old girl presents with progressive development of cavus feet.
What would be the most appropriate first line investigation when she is first seen by you in clinic?
A. Nerve conduction velocity studies
B. Biopsy of the quadriceps femoris muscle.
C. Biopsy of the sural nerve.
D. DNA testing.
E. Chromosomal analysis
Answer: A. Nerve conduction velocity studies
- Charcot-Marie-Tooth Disease, also known as peroneal muscular atrophy, is a common autosomal dominant hereditary motor sensory neuropathy, caused by abnormal peripheral myelin protein, that presents with muscles weakness and sensory changes which can lead to cavovarus feet, scoliosis, and claw foot deformities.
- Diagnosis is made with nerve conduction studies showing low nerve conduction velocities with prolonged distal latencies in the peroneal, ulnar, and median nerves.
- Treatment involves a multidisciplinary approach to address neuropathy, cavovarus and claw foot deformities, and scoliosis.
- Pathophysiology
o HMSN Type I
abnormal myelin sheath protein is the basis of this degenerative neuropathy.
results in a combination of motor and sensory disturbances.
o HSMN Type 2
intact myelin sheath with wallerian axonal degeneration that results in mild sensory and motor conduction velocities.
o pathoanatomy
peroneus brevis: peroneal involvement is typically first and most profound, results in muscle imbalance and varus deformity
tibialis anterior: weakness results in dropfoot
intrinsic muscles of hand and foot - check for wasting of 1st dorsal interosseous in hands - Genetics
o autosomal dominant duplication of chromosome 17 (most common): codes for peripheral myelin protein 22 (PMP 22) expressed in Schwann cells (most common) or X-linked connexin 32. But may also be autosomal recessive or X-linked. - Orthopedic manifestations: pes cavovarus, claw toes, hip dysplasia, Scoliosis, hand muscle atrophy and weakness
- Peroneus longus (more normal) overpowering weak tibialis anterior and weak intrinsics and contracted plantar fascia
- Varus caused by tibialis posterior (normal) overpowering weak peroneus brevis
Symptoms
* motor deficits: initial symptoms are distal weakness and atrophy of the distal muscles, instability during gait, clumsiness, frequent ankle sprains, difficulty climbing stairs
* lateral foot pain
* sensory deficits are variable
Physical exam
* Cavovarus foot: (similar to Freidreich’s ataxia) with hammer toes or clawing of toes, usually bilaterally and symmetric. Occurs due to unoposed pull of peroneus longus causing plantarflexion of the first ray and compensatory hindfoot varus. Initially flexible, but progresses to a rigid deformity
* motor weakness
o peroneal weakness: weakest muscles around foot and ankle
o anterior tibialis: weakens next, but typically stronger than the peroneals - can lead to drop foot in swing initially and later to a fixed equinus
o posterior tibialis: stays strong for a prolonged period of time
o weak intrinsics - including weak EDB and EHB
o clawtoes
* hyporeflexia or areflexia
* Coleman block test: test to determine if hindfoot varus deformity is secondary to plantar-flexed first ray vs an independent component.
o If deformity corrects with Coleman block, this suggests a forefoot driven varus deformity.
o If deformity does not correct with Coleman block, this suggests hindfoot driven varus deformity.
o a rigid hindfoot will not correct into neutral
* upper extremity: intrinsic wasting of hands, weak pinch and grasp
* spine: scoliosis may be evident on Adam’s forward bend test
- NCS: low nerve conduction velocities with prolonged distal latencies are noted in peroneal, ulnar, and median nerves. Can also see low amplitude nerve potentials due to axonal loss
- Genetic Testing: key component for diagnosis of CMT
o DNA analysis - PCR analysis used to detect peripheral myelin protein 22 (PMP22) gene mutations
o chromosomal analysis: duplication on chromosome 17 seen in autosomal dominant (most common) form
14.
You are in the Ponseti clinic treating a 3-week old boy. The stages in correction of the clubfoot deformity proceed in the following order:
A. Elevation of the first ray to correct cavus, abduction, then dorsi-flexion with a Tendo-achilles tenotomy
B. Pronation, dorsiflexion, abduction, Tendo-achilles tenotomy
C. Correction of varus, then supination then dorsiflexion with a Tendoachilles tenotomy
D. Elevation of the first ray to correct cavus, then dorsiflexion with a TA tenotomy followed by a final abduction cast
E. Elevation of the first ray to correct cavus, then dorsiflexion with a TA tenotomy
Answer: A. Elevation of the first ray to correct cavus, abduction, then dorsi-flexion with a Tendo-achilles tenotomy
CAVE
Clubfoot, also known as congenital talipes equinovarus, is a common idiopathic deformity of the foot that presents in neonates.
Diagnosis is made clinically with a resting equinovarus deformity of the foot.
Half of cases are bilateral and in 80%, clubfoot is an isolated deformity.
Genetic component is strongly suggested - recent link to PITX1.
Associated with arthrogryposis.
Pathophysiology
Muscle contractures contribute to the characteristic deformity that includes (CAVE):
- Cavus (tight intrinsics, FHL, FDL)
- Adductus of forefoot (tight tibialis posterior)
- Varus (tight tendoachilles, tibialis posterior, tibialis anterior)
- Equinus (tight tendoachilles)
Bony deformity consists of medial spin of the midfoot and forefoot relative to the hindfoot
- talar neck is medially and plantarly deviated
- calcaneus is in varus and rotated medially around talus
- navicular and cuboid are displaced medially
15.
You are asked to review a 7-year-old boy in A&E with early Perthes disease. Which of the following features would you expect to see on Pelvic X-Rays?
A. Bilateral symmetrical changes
B. Early acetabular changes
C. Irregular delayed proximal femoral ossification centres
D. Smaller sclerotic epiphysis with medial joint space widening
E. Femoral head deformity with widening and flattening
Answer: D. Smaller sclerotic epiphysis with medial joint space widening
XR findings:
Earliest findings: joint space widening (due to less ossification of head), irregularity of femoral head ossification, crescent sign (subchondral fracture).
Other investigations include MRI for earlier diagnosis, and an arthrogram to assess coverage and containment of the femoral head to help guide potential management.
Histology: femoral epiphysis and physis show areas disorganized cartilage, with areas of hypercellularity and fibrillation.
Legg-Calve-Perthes Disease is an idiopathic avascular necrosis of the proximal femoral epiphysis in children.
Diagnosis can be suspected with hip radiographs. MRI may be required for diagnosis of occult or early disease.
12% of cases are bilateral – usually asynchronous asymmetrical involvement – if symmetrical then is suggestive of multiple epiphyseal dysplasia.
Treatment is typically observation in children less than 8 years of age, with analgesia, activity modification and physio to maintain/improve ROM, and consideration for proximal femoral or acetabular osteotomy if required for containment.
Proximal Femoral Varus Osteotomy: for extrusion in early stages – aims to reposition femoral head into the acetabulum for containment purposes.
Valgus osteotomy considered if hinge abduction – lateral extrusion of the capital femoral epiphysis producing a painful hinge affect on the lateral acetabulum during abduction.
Shelf/Chiari osteotomies when femoral head is no longer containable.
Osteonecrosis occurs secondary to disruption of blood supply to femoral head, followed by revascularization with subsequent resorption and later collapse. Remodelling by creeping substitution after collapse. Repeated subclinical trauma and mechanical overload lead to bone collapse and repair. Damage results from epiphyseal bone resorption, collapse and the effect of subsequent repair during the course of disease.
Associated with coagulopathy, maternal/passive smoking, ADHD.
Lateral Pillar Classification – prognostic – patient needs to have entered the fragmentation stage radiographically.
Group A: lateral pillar maintains full height, no density changes identified – consistently good outcome.
Group B: Maintains >50% height, poor outcome if bone age >6years
Group B/C: Lateral pillar is narrowed (2-3mm) or poorly ossified with approximately 50% height.
Group C: Less than 50% of lateral pillar height is maintained – poor outcomes in all patients.
Waldenstrom Classification (Stages)
Initial – infacrtion produces a smaller, sclerotic epiphysis with medial joint space widening (XR may be occult for 3-6 months)
Fragmentation – begins with presence of subchondral lucent line (crescent sign). Femoral head appears to fragment/dissolve as a result of revascularisation process with bone resorption producing collapse with subsequent patchy density and lucencies. Lasts 6months – 2years. Symptoms most prevalent in this stage.
Reossification: ossific nucleus undergoes re-ossification, with new bone appearing as necrotic bone is resorbed. May last up to 18 months.
Remodelling: femoral head remodels until skeletal maturity – begins once ossific nucleus is completely reossified – trabecular pattern returns.
Caterall at risk signs with poor outcome: Gage sign (V-shaped lucency in lateral portion of the epiphysis and/or adjacent metaphysis), calcification lateral to the epiphysis, metaphyseal cyst, lateral subluxation of the femoral head, horizontal proximal femoral physis.
Stulberg Classification – for rating residual femoral head deformity and joint congruence.
Coxa magna = widened femoral head
Coxa plana = flattened femoral head
Presents with insidious onset intermittent hip, knee, groin or thigh pain, hip stiffness – loss of IR and abduction, altered gait (antalgic/Trendelenberg – decreased abductor tension), LLD is late
Good prognostic indicators: younger bone age (<6years @ presentation), spherity of femoral head and congruency at skeletal maturity (Stulberg classification), lateral pillar classification.
Poor prognosis: females, decreased hip abduction, heavy patient, longer duration from onset to complete healing, stiffness with progressive loss of ROM, Catterall at risk signs.
16.
You are in theatre performing a pelvic osteotomy on a 7-year-old. Which of the following is a salvage osteotomy?
A. Dega
B. Salter Innominate
C. Shelf
D. Pemberton
E. Triple
Answer C. Shelf
periacetabular osteotomy (PAO) for symptomatic dysplasia in an adolescent or adult with a concentrically reduced hip and congruous joint space with a preserved range of motion. Triradiate cartilage must be closed.
Intraoperative dynamic testing of hip motion is needed to determine the need for femoral osteotomy (minimum of 90° flexion and 15° internal rotation to prevent FAI)
Multiple osteomies in pubis, ilium and ischium near the acetabulum, technically most challenging.
Advantages:
provides hyaline cartilage coverage
preserved integrity of the posterior column, which allows patients to weight bear as tolerated postoperatively (posterior column and pelvic ring remain intact).
large multidirectional corrections
preserves external rotators
delays need for arthroplasty
outcomes
reliably improves radiographic parameters and symptomatology
92% survivorship at 15 years in avoiding THA
Chiari = salvage osteotomy for unreduced hip - recommended for patients with inadequate femoral head coverage and incongruous joint (concentric reduction cannot be obtained) - Osteotomy starts above the acetabulum to the sciatic notch and ileum is shifted lateral beyond the edge of the acetabulum.
Depends on fibrocartilage
metaplasia for successful results.
Medializes the acetabulum via iliac osteotomy
Shelf: salvage in patients >8 years, add bone to the lateral weight bearing aspect of the acetabulum by placing extra-articular buttress of bone over the subluxed femoral head.
Depends on fibrocartilage metaplasia for successful results.
Salter - redirectional: younger patient with open triradiate cartilage - single transverse cut above the acetabulum through the ilium to sciatic notch. Acetabulum then hinges through pubis symphysis - improves anterolateral coverage (may length leg up to 1cm)
Triple (Steele) is Salter with additional cuts through superior and inferior rami - redirectional for anterolateral coverage)
Dega - volume reducing. favoured in neuromuscular dislocation and patients with posterior acetabular deficiency - for severe cases.
Osteotomy from acetabular roof to triradiate cartilage (incomplete cuts through peri-capsular of the innominate bone).
Acetabulum hinges through the triradiate cartilage. Does not enter the sciatic notch and is therefore stable and does not require internal fixation.
Improves anterior, central or posterior coverage.
Pemberton = volume reducing, moderate to severe. Triradiate cartilage must be open. stable as osteotomy does not enter sciatic notch.
17.
You are performing an arthrogram on a child. On an arthrogram, which one of the following is NOT seen in DDH?
A. A ‘rosethorn’ appearance on the antero‐superior aspect of the joint.
B. Inverted labrum
C. Pooling of the contrast
D. Gage’s sign
E. Hourglass constriction
Answer: D. Gage’s sign
Gage’s sign is seen in Perthes Disease – not DDH: Caterall at risk signs with poor outcome: Gage sign (V-shaped lucency in lateral portion of the epiphysis and/or adjacent metaphysis)
Dysplasia leads to subluxation and gradual dislocation, repetitive subluxation of the femoral head leads to the formation of a ridge of thickened articular cartilage called the limbus.
Chronic dislocation leads to the development of secondary barriers to reduction: pulvinar thickens, ligamentum teres thickens and elongates, TAL hypertrophies, hip capsule and iliopsoas form hourglass configuration. Anatomic changes include increased femoral anteversion, flattening of the femoral head, increased acetabular anteversion, increased obliquitiy and decreased concavity of the acetabular roof, thickening of the medial acetabular wall.
Developmental Dysplasia of the Hip is a disorder of abnormal development resulting in dysplasia, subluxation, and possible dislocation of the hip secondary to capsular laxity and mechanical instability.
Diagnosis can be confirmed with ultrasonography in the first 4 months and then with radiographs after femoral head ossification occurs (~ 4-6 months).
Treatment varies from Pavlik bracing to surgical reduction and osteotomies depending on the age of the patient, underlying etiology, and the severity of dysplasia.
Risk factors: firstborn, female, breech, packing disorders, family history, macrosomia, limited hip abduction, talipes, swaddling
Dysplasia: shallow or underdeveloped acetabulum
Subluxation: displacement of the joint with some contact remaining between the articular surfaces
Dislocation: complete displacement of the joint with no contact between the original articular surfaces
Teratologic hip: dislocated in utero and irreducible on neonatal exam, presents with a pseudoacetabulum - associated with neuromuscular conditions and genetic disorders. Commonly seen with arthrogryposis, myelomeningocele, Larsen’s syndrome, Ehlers-Danlos
Late (adolescent) dysplasia: mechanically stable and reduced but dysplastic
Typical deficiency is anterior or anterolateral acetabulum, BUT in spastic cerebral palsy acetabular deficiency is posterosuperior – needs to be considered when planning pelvic osteotomies!
Associated with other packing deformities: congenital muscular torticollis (20%), metatarsus adductus (10%), congenital knee dislocation.
Barlow: dislocates a dislocatable hip by adduction and depression of a flexed femur – ‘click of exit”
Ortolani: reduces a dislocated hip by elevation and abduction of the flexed femur – ‘click of entry’
Galeazzi (Allis): apparent limb length discrepancy due to unilateral dislocated hip with hip flexed at 90 degrees and feet on the table – femur appears shortened on dislocated side.
Klisic test: for bilateral dislocations: line from long finger placed over the greater trochanter and the index finger over the ASIS should point to the umbilicus – if the hip is dislocated, the line will point halfway between the umbilicus and pubis.
18.
A 5‐year‐old boy presents with a limp and pain over the medial aspect of the foot. He is diagnosed with Kohler’s disease.
Orthotics have failed to improve his symptoms. What is the next appropriate management of this condition?
A. Core decompression
B. Triple arthrodesis
C. Subtalar fusion
D. IV antibiotics with irrigation and debridement
E. Symptomatic treatment with analgesia/short period of rest in cast/orthotica
Answer: E. Symptomatic treatment with analgesia/short period of rest in cast/orthotica
- Kohler’s Disease is a rare idiopathic condition caused by avascular necrosis of the navicular bone that occurs in young children and presents with pain on the dorsal and medial surface of the foot. More common in boys, aged 4-7 years, can be bilateral in up to 25% of cases. Presents with mid-foot pain and a limp, may be asymptomatic. May have swelling, warmth & redness with point tenderness over navicular.
- Diagnosis is made with radiographs of the foot showing sclerosis, fragmentation, and flattening of tarsal navicular bone – most re-organise after disease has run it’s course, but even when continue to be deformed almost all are asymptomatic.
- Treatment is usually nonoperative with NSAIDs and a short period of cast immobilization as the condition typically resolves over time. XRs typically improve 6-48 months after onset, with no reports of long-term disability.
Surgery is not indicated – typically self limiting, intermittent symptoms may remain for 1-3 years after diagnosis.
Blood supply to the central third of the navicular is a watershed zone – accounting for the susceptibility to avascular necrosis and stress fractures. The navicular is the last bone to ossify (therefore increased vulnerability to mechanical compression and injury).
19.
A 20-year-old man attends clinic following an MRI which has confirmed a diagnosis of Kienbock’s disease.
Which of the following is NOT thought to contribute to Kienbock’s disease?
A. Ulnar negative variance
B. Lunate geometry
C. Ulnar positive variance
D. Vascular anatomy of lunate
E. Decreased radial inclination
Answer: C. Ulnar positive variance
Kienbock’s Disease is the avascular necrosis of the lunate which can lead to progressive wrist pain and abnormal carpal motion. Most commonly affecting males 20-40years old.
Diagnosis can be made with wrist radiographs in advanced cases but may require MRI for detection of early disease.
Treatment is NSAIDs and observation in minimally symptomatic patients. A variety of operative procedures are available depending on severity of disease and patient’s symptoms.
Risk factors:
- Ulnar negative variance (leads to increased radial-lunate contact area)
- Decreased radial inclination
- Repetitive trauma
- Lunate geometry
- Vascular supply to lunate – patterns of arterial blood supply (3 patterns: Y-pattern, X-pattern, I-pattern (highest risk AVN)) have differential incidence of AVN – disruption of venous outflow leading to increased intraosseous pressure. Blood supply to capitate is also poor and may lead to AVN.
Presents with dorsal wrist pain usually activity related, may be swelling, decreased flexion/extension arc, decreased grip strength.
MRI is best for diagnosis of early disease and rules out ulnar impaction.
CT is most useful once lunate collapse has already occurred – for showing extent of necrosis, trabecular destruction and lunate geometry.
Lichtman Classification:
Stage I: changes on MRI only – immobilisation & NSAIDs
Stage II: Sclerosis of lunate – consider joint levelling procedure (in ulnar negative patients), radial wedge osteotomy or STT fusion (in ulnar neutral patients), distal radius core decompression, revascularisation procedures.
Stage IIIA: Lunate collapse, no scaphoid rotation – same treatment as Stage II
Stage IIIB: lunate collapse, fixed scaphoid rotation – proximal row carpectomy, STT fusion or SC fusion
Stage IV: degenerated adjacent intercarpal joints – wrist fusion, PRC, limited intercarpal fusion.
20.
A 25-year-old man has presented with a radial head fracture that you have decided is for surgical fixation. When planning for surgery you decide to use the Kaplan interval for your approach.
Which is the intermuscular interval that you will use?
A. Brachioradialis and ECRL
B. Anconeus and ECU
C. ECU and EDC
D. ECRB and EDC
E. ECRB and ECRL
Answer: D. ECRB and EDC
Kocher: ECU (PIN) and anconeus (radial N) - palpate the radial head and lateral epicondyle - 5cm oblique posterolateral incision in line. Need to repair LUCL.
Kaplan: EDC (PIN) and ECRB (radial N) more anterior therefore more caution re: PIN
Anteromedial Hotchkiss approach:
- Mark out medial epicondyle, ulnar nerve and olecranon ridge
- 10cm curving incision from medial epicondyle
- Internervous plane: FCU (ulnar nerve) and FCR(medial nerve)/PL
- Identify medial supracondylar ridge of humerus, peel of brachialias, extend interval down to capsule and coronoid.
Radial head fracture - Mason (modified by Hotchkiss and Broberg-Morley) 1-4: 1- non/minimally displaced, no mechanical block to rotation, II - displaced > 2mm or angulation, possible mechanical block to forearm rotation, III - comminuted and displaced, mechanical block to rotation, IV - radial head fracture with associated elbow dislocation. (Usually a posterolateral dislocation)
O’Driscoll’s Classification:
Elbow Stabilisers:
Primary: MCL, LCL, ulnohumeral joint
Secondary: flexor and extensor muscles, radial head, capsule
O’Driscoll terrible triad: radial head, coronoid, fractures, elbow dislocation
21.
A 30-year-old lady is seen in a follow up clinic complaining that she is only able to fully flex one finger following an operation to repair a structure in her hand following a glass laceration.
Which structure has been repaired?
A. Flexor digitorum profundus
B. Flexor digitorum superficialis
C. Flexor pollicis longus
D. Extensor pollicis longus
E. Extensor digitorum communis
Answer: A. Flexor digitorum profundus
22.
A Martin-Gruber anastomosis is:
A. Median nerve to ulnar nerve in the forearm
B. Ring finger to middle finger common digital nerve in the hand
C. Between recurrent branch median and deep motor branch ulnar in the hand
D. Ulnar nerve branches into medial nerve in the forearm
E. Ulnar nerve to musculocutaneous nerve in the arm
Answer: A. Median nerve to ulnar nerve in the forearm
Marti-Gruber anastomosis: communicating nerve crossing over from the median to ulnar nerve in the forearm; (motor connections but not sensory connections). Can cause confusion both clinically and on EMG. Incidence around 15%
it occurs in two patterns:
- from median nerve in proximal forearm to ulnar nerve in middle to distal third of forearm, & from AIN to ulnar nerve, innervating the intrinsic muscles in the hand.
Occurs more commonly on the right side and can be Autosomal dominant.
A lesion of the median nerve above the communicating branch will affect the median nerve muscles.
A lesion of the ulnar nerve below the anastomosis will not affect the median nerve muscles, it will spare the thenar motor intrinsics.
Isolated ulnar nerve lesion at the elbow unusual pattern of intrinsic muscle paralysis.
Damage to the ulnar nerve at the wrist more severe deficit of intrinsics and hand function than expected.
Marinacci anastomosis: ulnar to median in forearm (reverse of Martin-Gruber)
Riche-Cannieu anastomosis: ulnar to median anastomosis in the hand – connection between the deep branch of ulnar nerve and recurrent branch of the median nerve. It carries motor fibres and the anastomosis usually occurs in the region of the thenar and adductor pollicis muscles.
Berrettini anastomosis: communication between the digital nerves (sensory!) arising from the ulnar and median nerves in the hand. Most common nerve anastomosis pattern.
23.
In which order are the following articulations disrupted in a lesser arc perilunate injury, as established by Mayfield?
A. Scapholunate - Lunotriquetral -Capitolunate - Radiocarpal
B. Radiocarpal - Scapholunate - Lunotriquetral -Capitolunate
C. Scapholunate - Capitolunate - Lunotriquetral - Radiocarpal
D. Capitolunate - Lunotriquetral - Scapholunate - Radiocarpal
E. Capitolunate - Lunotriquetral - Radiocarpal - Scapholunate
Answer: C. Scapholunate - Capitolunate - Lunotriquetral - Radiocarpal
- Sequence of events:
o scapholunate ligament disrupted –>
o disruption of capitolunate articulation –>
o disruption of lunotriquetral articulation –>
o failure of dorsal radiocarpal ligament –>
o lunate rotates and dislocates, usually into carpal tunnel
Peri-lunate Injuries:
- Piece of pie sign, spilled tea cup sign, Gilulla’s lines
- Dislocation of lunate from carpus – is a rare, high energy injury that involves ligamentous +/- boney components, and can -> complex carpal instability, degenerative change, decreased range of movement, grip strength.
- Devastating injury but innocuous on XR, can be easily missed therefore important to always get an AP and a lateral.
- 25% are missed at presentation – delayed reconstruction can -> poor outcomes
- Gilula’s lines:
o Proximal edge of distal carpal bones
o Distal edge proximal carpal bones
Proximal edge proximal carpal bones
o
Mayfield classification:
- Originally purely soft tissue
- A predictable pattern of injury occurs:
o Hyperextended wrist is taken in ulnar deviation
o Capitate becomes the centre of rotation -> injury to soft tissue on the radial side of the wrist occurs first
o As the forces progresses through the carpus -> intercarpal supination which is centred through the triquetrum -> injury to the soft tissue on the ulnar side of the wrist:
1. Scapholunate ligament injury
2. Scapholunate ligament injury + midcarpal ligament injury
3. Scapholunate ligament injury + midcarpal ligament injury + lunotriquetral ligament injury - “Classic” Perilunate
4. Injury to radiolunate ligaments – associated with median nerve compression
Johnson Classification:
Lesser Arc injuries: purely ligamentous
Greater Arc: ligamentous disruptions with associated fractures of the radius, ulnar, or carpal bones – bone involvement = ‘trans’ injury
Ligaments:
- Intrinsic: Origin and insertion within the same row e..g scapholunate, lunotriquetral
- Extrinsic:
o Palmar ligaments: 2 parallel chevrons, between which is an area of relative weakness (space of Poirier – this is the area through which the lunate dislocates in a Mayfield 4) – radioscaphocapitate (distal), long and short radiolunate (proximal), Ulnalunate ligament
o Dorsal ligaments: (Berger’s approach)
Proximal limb: dorsoradiocarpal ligament
Distal limb: dorsal intercarpal ligament
Carpal Instability – 4 main types
1. Adaptive carpal instability (post distal radius fracture non-union)
2. Carpal instability dissociative – between bones in the same row, most commonly proximal row (scapholunate/lunotriquetral)
3. Carpal instability NON-dissociative – instability between the proximal and distal rows – i.e. the rows themselves remain well aligned.
4. Complex carpal instability = combination of 2&3 (Mayfield 2,3 & 4)
Scapholunate syndrome = trans-scaphoid, transcapitate therefore proximal pole of the capitate can rotate up to 180 degrees, leading to significant mid carpal degeneration
Assessment:
Emergent Management: Assess for any median nerve symptoms! Reduce the carpus – use your thumb to support the lunate whilst hyperextending, then flexing. 20-30mins of fingertraps can aide ligamentotaxis.
CT scan – then refer to local hand team.
If you take to theatre to reduce then you MUST also decompress the median nerve – won’t know if you have caused median nerve compression on reduction while patient anaesthetised!
If closed reduction is unsuccessful:
- Mayfield 4 -> palmar approach
- Mayfield 3 -> palmar +/- dorsal approach
Principles of definitive management:
1. Reduce the lunate
2. Hold reduced
3. Reconstruct injured structures
4. Protect repairs until healed
Palmar approach – release transverse carpal ligament to decompress median nerve and reduce the lunate
Dorsal Approach: landmarks = Lister’s tubercle, EPL
- Longitudinal midline dorsal incision (used interval between 3rd and 4th extensor compartments – flex the common extensors away)
Berger/Mayo flap:
- K-wire: 1x into distal pole of scaphoid and 1x into lunate (converging) – site the wires parallel to the concavity of the lunate (in normal reduced anatomy this wire should then be at 90degrees so acts to aide fracture reduction. Then bend wires towards eachother and this will reduce the scapho-lunate joint.
- Then use a scapholunate wire to protect the SLL, triquetrolunate wire to protect lunotriquetral ligament, then scaphocapitate and triquetrohamate to stabilise mid-carpal row.
Then move on to soft tissue reconstruction e.g. suture anchors/Miteks etc
Greater arc injuries: fix bone 1st, work proximal to distal i.e. radius then scaphoid, then reconstruct ligaments.
Outcomes: 75% range of movement, 75% grip strength (according to retrospective 10 year follow up – Forli et al)
24.
Which of these structures is NOT part of the triangular fibrocartilage complex?
A. Meniscus homolog
B. ECU subsheath
C. Radioscaphocapitate ligament
D. Ulnotriquetral ligament
E. Radioulnar ligament
Answer: C. Radioscaphocapitate ligament
TFCC is made up of:
- Dorsal and volar radioulnar ligaments: deep ligaments, known as ligamentum subcruetum, which insert into the ulnar fovea, and superficial fibres that insert in the ulnar styloid. They originate at the sigmoid notch of the radius and converge at the base of the ulna styloid.
- Central articular disc
- Meniscus homolog
- Ulna collateral ligament
- ECU subsheath
- Origin of ulnolunate and ulnotriquetral ligaments
The periphery is well vascularised, whereas the central portion is avascular.
TFCC injury is a common cause of ulnar-sided wrist pain, diagnosis is pain worse on ulnar deviation and a positive ‘fovea’ sign, MRI can confirm the diagnosis, although arthroscopy remains the most accurate method of diagnosis.
Type 1 traumatic injury: most common mechanism is a fall on extended wrist with forearm pronation or a traction injury to the ulnar wrist.
Type 2 degenerative injury: associated with positive ulnar variance and ulnocarpal impaction.
Treatment is generally conservative (NSAIDS and immobilisation). Surgical debridement, TFCC repair or ulnar shortening procedures may be indicated depending on severity of symptoms and underlying cause.
25.
Which of the following is the correct flexor pulley of the hand that is affected in trigger finger (not thumb)?
A. A3
B. C1
C. C3
D. A1
E. A2
Answer: D. A1
A1 pulley overlies the MPJs
Trigger Finger (trigger thumb when involving the thumb) is the inhibition of smooth tendon gliding due to mechanical impingement at the level of the A1 pulley that causes progressive pain, clicking, catching, and locking of the digit.
Diagnosis is made by physical examination with presence of active triggering and tenderness at the A1 pulley.
Treatment consists of splinting, anti-inflammatory medications, steroid injections, and surgical release (percutaneous A1 pulley release – success rate of >90%). Higher recurrence in diabetic patients. Open release is easier to assess quality of release compared to percutaneous, success >90%.
If persistent/recurrent triggering after A1 pulley release considered also releasing a slip of FDS (usually ulnar slip).
Rheumatoid arthritis may benefit from FDS slip excision without A1 pulley release – sparing of the A1 pulley may prevent exacerbation of ulnar drift at the MCPJ.
N.B. Paediatric trigger finger presents with Notta’s node (proximal to A1 pulley), flexion contracture an triggering. Surgical treatment usually around 2-4 years to prevenet interphalangeal joint contracture with release of A1 pulley and 1 slip of FDS – may even need to release the remaining FDS slip and A3 pulley. Success rate >90%.
More common in diabetics and females over 50 years.
Ring and long finger are most commonly involved in adults.
Pathophysiology: stenosing tenosynovitis at the A1 pulley due to fibrocartilaginous metaplasia of the tendon and/or pulley, proliferation of chondrocytes, increased type III collagen. Chronic hyperglycaemia creates collagen cross-links and impairs collagen degradation.
Pathoanatomy: Occasional pathologic nodule of FDP tendon. FDS often unaffected.
Trigger thumb may have a 4th pulley (variable annular pulley) causing stenosis in up to 75% patients.
Associations: rheumatoid arthritis, calcific tendinitis, carpal tunnel syndrome (>60% of trigger digits have clinical or electrodiagnostic evidence of carpal tunnel syndrome), congenital trigger thumb, diabetes (more commonly bilateral and multiple digit), amyloidosis, hypothyroidism, sarcoidosis, gout, pseudogout.
Pathologic nodule that may develop on FDP inhibits the smooth tendon gliding through the A1 pulley.
Usually progressive – pain at level A1 pulley, clicking, catching, finger ofter becomes locked in a flexed position at the PIPJ.
26.
Which of the following is true about Flexor Zone III of the hand?
A. Distal extension is the transverse palmar crease
B. Lacerations in this zone are often accompanied by median nerve injury
C. Proximal boundary is the proximal end of the transverse retinacular ligament
D. Tendons in this zone are free of a tendon sheath
E. Tendon receives its vascular supply through vincula
Answer: A. Distal extension is the transverse palmar crease
Verden Zones:
I: Distal to FDS insertion (Jersey finger)
II: FDS insertion to distal palmar crease/proximal A1 pulley – Zone is unique inthat the FDS and FDP are in the same tendon sheath – so both can be injured within the flexor retinaculum. Tendons can retract if the vincula are disrupted. Tx: Direct tendon repair with early ROM. Previously poor outcomes, but improving with newer post-op protocols.
III: Palm (A1 pulley to distal aspect of carpal ligament) – often associated with NV injury, which carries a worse prognosis. Tx: Direct tendon repair, generally good results due to absence of retinacular structures. May require A1 pulley release to avoid impingement of the repaired tendon on the pulley.
IV: Carpal tunnel – often complicated by post-op adhesions due to closeness of synovial sheath and carpal tunnel. Tx: direct tendon repair. Transverse carpal ligament should be repaired in a lengthened fashion if tendon bowstringing is present.
V: Carpal tunnel to forearm - often associated with NV injuries – which carry as worse prognosis.
Tendons receive blood supply via 2 sources:
- Diffusion through synovial sheaths: occurs when flexor tendons are located within a sheath and is the more important source distal to the MCP joint
- Direct vascular perfusion nourishes flexor tendons located outside of synovial sheaths. Supplied by the vincular system, osseous bony insertions, reflected vessels from the tendon sheath, and longitudinal vessels from the palm
The vascularity of tendon varies depending on the type of tendon (e.g. with or without a sheath) and the location. Sheathed tendons (e.g. flexor tendons of the hand) have a dual blood supply via both vascular perfusion but also have regions that are relatively avascular where they receive nutrition through synovial diffusion. This is the case in zone 2 of the digital flexor tendons where the primary nutritional supply is from synovial diffusion through the parietal paratenon which allows for passive nutrient delivery to the flexor tendon within the sheath. The digital flexor tendons also receive minor direct arterial perfusion in zone 2 through the vinicular system, osseous bony insertions, reflected vessels from the tendon sheath and longitudinal vessels from the palm, but this is not the major blood supply.
Tendons not enclosed by a sheath receive their blood supply directly from vessels entering from the tendon surface or from the tendon-to-bone insertion.
Flexor Tendon Injuries are traumatic injuries to the flexor digitorum superficialis and flexor digitorum profundus tendons that can be caused by laceration or trauma.
Diagnosis is made clinically by observing the resting posture of the hand to assess the digital cascade and the absence of the tenodesis effect. Treatment is usually direct end-to-end tendon repair.
Tendon healing - occurs via 2 pathways: (3 phases - inflammatory, fibroblastic, remodelling)
- Intrinsic: produced by tenocytes within the tendon
- Extrinsic: stimulated by surrounding synovial fluid and inflammatory cells. Implicated in the formation of scarring and adhesions
Camper chiasm: located at the level of the proximal phalanx where FDP splits FDS
Pulley system:
Digits 2-5:
- 5 annular pulleys (A1 to A5) which are thicker and stiffer than cruciate pulleys. A2 and A4 arise from the periosteum and are the most important pulleys to prevent flexor tendon bowstringing. A1, A3, and A5 arise from the volar plate
- 3 cruciate pulleys (C1 to C3) : collapsible and flexible - allows the annular pulleys to approximate each other during digital flexion
Whereas thumb contains 3 annular pulleys (A1, Av, A2) (A2 contributes least to arc of motion of thumb) and 1 interposed oblique pulley (most important pulley to prevent flexor tendon bowstringing (along with A1 pulley))
Indications for Flexor tendon repair:
> 75% laceration or ≥ 50-60% laceration with triggering (for these epitendinous suture at the laceration site is sufficient - no benefit of adding core suture)
Fundamentals of repair
- Easy placement of sutures in the tendon
- Secure suture knots
- Smooth juncture of the tendon ends
- Minimal gapping at the repair site
- Minimal interference with tendon vascularity
- Sufficient strength throughout healing to permit application of early motion stress to the tendon
Perform repair within three weeks of injury (2 weeks is ideal) - delayed treatment leads to difficulty due to tendon retraction
Approach: incisions should always cross flexion creases transversely or obliquely to avoid contractures (never longitudinal). Meticulous atraumatic tendon handling minimizes adhesions
27.
The most common method of performing a z‐plasty in hand surgery theoretically yields 75% of lengthening along the line of the central limb.
Which method should be used to achieve this?
A. 45‐degree angle between the limbs with one limb 10% longer than the other
B. 45‐degree angle between the limbs of equal length
C. 60‐degree angle between the limbs with one limb 10% longer than the other
D. 60‐degree angle between the limbs of equal length
E. 90‐degree angle between the limbs of equal length
Answer: D. 60‐degree angle between the limbs of equal length
One of the most commonly used techniques for lengthening scar contracture in hand surgery is the Z-plasty. When the two 60 degree triangular flaps are transposed and closed, the original direction of the scar is rotated and the scar length is increased by approximately 75% Because of its history the 60 degree Z-plasty is the technique to which other methods of contracture lengthening are compared.
28.
A 65-year-old man presents with Dupuytren’s contractures affecting the 4th and 5th digits of his right hand (15 degrees at interphalangeal joints and 30 degrees at metacarpophalangeal joints) and seeks surgery.
Which surgical procedure would be most appropriate?
A. Subcutaneous fasciotomy
B. Partial/selective fasciectomy
C. Complete fasciectomy
D. Complete fasciectomy with skin grafting
E. Amputation
Answer: B. Partial/selective fasciectomy
Dupuytren’s Disease is a benign proliferative disorder characterized by decreased hand function caused by hand contractures and painful fascial nodules.
Diagnosis can be made by physical examination which shows painful nodules in the palm with associated digital contracture.
Treatment ranges from nonoperative passive stretching to injections, needle aponeurotomy, and operative open fasciectomy if the disease progresses or affects a patient’s daily living.
More severe disease in men than women, with a 2:1 ratio M:F, most commonly presenting in 5-7th decade life. Autosomal dominant with variable penetrance.
Ring >small>middle>index
Myofibroblast is the dominant cell type (differs from fibroblast as the myofibroblast has INTRACELLULAR ACTIN filaments aligned along long axis of cell). Adjacent myofibroblasts connect via EXTRACELLULAR FIBRONECTIN to act together to create contracted tissue.
Type III collagen predominates (>type I collagen), cytokines also implicated (TGF beta-1, -2, epidermal growth factor, PDGF, connective tissue growth factor.
Ectopic manifestations: Ledderhose disease (planta fascia) 10-30%, Peyronie’s disease (dartos fascia of penis) 2-8%, Garrod disease (knuckle pads) 40-50%
Associated with HIV, alcoholism, diabetes, antiseizure medications
Nodules and cords make up the pathologic anatomy – with nodules appearing before contractile cords.
Normal fascial bands become pathologic cords:
- Palmar pretendinous cord
- Palmodigital transition natatory cord, spiral cord
- Digital central cord (distal extent of the pre-tendinous cord), lateral cord, digital cord, retrovascular cord.
Spiral cord = most important cord – cause of PIP contracture, typically inserts distally into the lateral digital sheet, then into Grayson’s ligament – composed of pre-tendinous band, spiral band, lateral digital sheet and Grayson’s ligament.
Travels under the NV bundle, displacing it central and superficial – therefore at risk during surgical resection. Best predictors of displacement are PIPJ flexion contracture (77% PPV) and interdigital soft-tissue mass (71% PPV)
Central cord = from disease involving pretendinous band. Inserting into flexor sheath at PIPJ level and causes MCP contracture. Forms palmar nodules and pits between distal palmar crease and palmar digital crease – NOT involved with NV bundle.
Retrovascular cord = runs dorsal to the NV bundle distally, originates from proximal phalanx and inserts on distal phalanx. Causes DIP contracture
Natatory cord (from natatory ligament) causes web space contracture.
NOT involved in Dupuytren’s disease: Clelands ligament and transverse ligament of the palmar aponeurosis.
Stages: Proliferative, involutional, residual
Treatment is not indicated where there is no contracture, and in patients with a mild (<20deg) contracture, or one which is not progressing and does not impair function.
Intervention should be considered for finger contracutres causing a loss of finger extension of 30deg or more at MCP or 20deg at the PIP, or severe thumb contractures which interfere with function.
Needle aponeurectomy: mild contractures at MCP >PIP, higher recurrence rate and less improvement than open partial fasciectomy.
29.
In the surgical treatment of de Quervain’s stenosing tenosynovitis, which of the following tendons should be decompressed?
A. Abductor Pollicis Longus
B. Adductor Pollicis
C. Extensor Pollicis Longus
D. Flexor Pollicis Longus
E. Opponens Pollicis
Answer: A. Abductor Pollicis Longus
De Quervain’s Tenosynovitis is a stenosing tenosynovial inflammation of the 1st dorsal compartment.Most common in the dominant wrist, affecting women > men, 30-50 years.
Diagnosis is made clinically with radial sided wrist pain (1st dorsal compartment at level of radial styloid), made worse with the Finkelstein manouvre (grasp patients thumb and quickly abduct hand ulnarward – more indicative of EPB > APL pathology) and pain on resisted radial deviation.
Treatment is generally conservative with thumb spica braces, injections and in refractory cases, 1st dorsal compartment surgical release.
Due to thickening and swelling of the extensor retinaculum causing increased tendon friction – Not considered an inflammatory process.
Extensor compartments:
1: EPB, APL (De Quervain’s tenosynovitis)
2:ECRB, ECRL (Intersection syndrome)
3: EPL
4: Extensor indicis, EDigitorum
5: EDM (Vaughan-Jackson syndrome)
6: ECU (Snapping ECU)
Surgical release of 1st dorsal compartment if severe symptoms having failed 6 months of non-op treatment. Caution for superficial radial sensory nerve as radial based incision proximal to the wrist.
Transverse incision with release on dorsal side of 1st compartment to prevent volar subluxation of the tendon – EPB is more dorsal than APL. Has variable anatomy with APL usually having at least 2 tendon slips and its own fibro-osseous compartment. A distinct EPB sheath is often encountered dorsally.
Complications: injury sensory branch radial nerve, neuroma formation, failure to decompress with recurrence, CRPS.
High recurrence rate.
30.
Which of these structures is NOT a recognised contributor to compression of the ulnar nerve around the elbow?
A. Arcade of Struthers
B. Anconeous epitrochliaris muscle
C. Osborne’s ligament
D. Arcade of Osborne
E. Medial collateral ligament
Answer: E. Medial collateral ligament
Cubital Tunnel Syndrome is a compressive neuropathy of the ulnar nerve caused by anatomic compression in the medial elbow.
Diagnosis is made clinically with presence of sensory changes to the ring and little finger, intrinsic muscle weakness and a positive tinel’s sign over the cubital tunnel.
Treatment may be nonoperative modalities such as bracing or surgical decompression depending on the severity and duration of symptoms, and success of nonoperative treatment.
Sites of entrapment:
Most common:
- Between the 2 heads of FCU/aponeurosis (commonest)
- Within the arcade of Struthers
- Between Osbourne’s ligament and MCL
Other sites include:
- Medial head of triceps
- Medial intermuscular septum
- Fascial bands within FCU
- Anconeus epitrochlearis (anomalous muscle from the medial olecranon to the medial epicondyle)
- Aponeurosis of FDS proximal edge
External Sources of compression:
- Fractures and medial epicondyle non-unions
- Osteophytes
- Heterotophic ossification
- Tumours and ganglion cycsts
- Post-traumatic
Ulnar Nerve – medial cord brachial plexus (C8-T1). Lies posteromedial to brachial artery in anterior compartment of upper arm, pierces IM septum at arcade of Struthers 8 cm proximal to the medial epicondyle. Runs behind the medial epicondyle within the cubital tunnel. Enters the forearm between 2 heads (humeral and ulnar heads) of FCU, then runs between FCU and FDP, passes superficial to transverse carpal ligament at the wrist.
Cubital tunnel:
- Roof: formed by FCU fascia and Osbournes ligament (travels from medial epicondyle to the olecranon)
- Floor: formed by posterior oblique and transverse bands of MCL and elbow joint capsule
- Walls: formed by medial epicondyle and olecranon.
31.
Regarding the surgical management of thoracic outlet syndrome which of the following is NOT a recognised surgical technique for thoracic outlet decompression?
A. Scalenotomy
B. Scalenectomy
C. Neurolysis
D. Pectoralis major release
E. Claviculectomy
Answer: D. Pectoralis major release
Pectoralis MINOR – not major is implicated in thoracic outlet sydrome
Thoracic outlet syndrome is a neurovascular disorder resulting from compression of the brachial plexus and/or subclavian vessels in the interval between the neck and axilla.
Diagnosis can be suspected clinically with specific provocative tests and supplemented with radiographs or vascular studies. showing anatomic causes of compression.
Treatment may be nonoperative or include surgical decompression or a vascular procedure depending on the specific etiology.
neurogenic is most common (95%)
vascular may be venous (4%) or arterial (< 1%)
- hypertrophy of anterior scalene
- scalenus minimus: accessory muscle found in 30-50% of patients with TOS. Originates from cervical transverse process and inserts onto 1st rib between the subclavian artery and T1 root
-fibromuscular bands: increase stiffness and decrease compliance of the thoracic outlet
- costoclavicular ligament: abnormal insertion implicated in Paget-Schroetter syndrome (intermittent obstruction of subclavian vein in costoclavicular space –> upper extremity DVT)
- soft tissue tumors: Pancoast tumor, neuroblastoma, schwannoma brachial plexus
- abnormal pec minor
Osseous:
- cervical rib (arises from C7 vertebra)
- prominent C7 transverse process
- abnormal clavicle or 1st rib
- ACJ or SCJ injury or dislocation
- Osseous tumours - bone mets, osteoid osteoma
- Chronic overuse - repetitive lifting - weight lifters, rowers, swimmers
Thoracic outlet is composed of 3 distinct spaces:
- interscalene triangle: brachial plexus trunks, subclavian artery
- costoclavicular space: brachial plexus divisions, subclavian artery and vein
- retropectoralis minor space: brachial plexus cords, axillary artery and vein
32.
When approaching a proximal diaphyseal radius fracture via the Henry approach (volar), the forearm is supinated to minimise injury to what structure?
A. Ulnar nerve
B. Median nerve
C. Posterior interosseus nerve
D. Lateral antebrachial cutaneous nerve
E. Radial nerve
Answer: C. Posterior interosseus nerve
The Henry approach is the volar approach to the forearm. The internervous plane is pronator teres (median) and brachioradialis (radial nerve). The arm should be supinated to move the PIN away from the surgical field. Conversely, in the Thompson (posterior) approach to the forearm, the forearm should be pronated to move the PIN away from the surgical field.
Longitudinal incision just lateral to biceps tendon on flexor crease of elbow (can be extended all the way down to radial styloid process).
Incise the deep fascia in line with skin incision, develop plane between BR & FCR distally, and between PT and BR proximally, identify the superficial radial nerve beneath BR, ligate branches of the radial artery to aide lateral retraction of BR.
Deeper dissection: follow the biceps tendon to its insertion on the bicipital tuberosity. Incise the bursa radial to the insertion of biceps tendon to gain access to proximal part of radius (care as radial artery runs along the ulnar side of the biceps tendon).
Fully supinate the forearm to displace the PIN radially and bring the origin of the supinator muscle into the anterior aspect of the radius.
Incise supinator muscle along the line of it’s broad insertion and continue subperiosteal dissection laterally.
Posterior interosseous nerve enters the supinator muscle beneath a fibrous arch known as the arcade of Frohse - the arch is formed by the thickened edge of the superficial head of the supinator muscle. Compression of the nerve at this point produces paralysis or dysfunction of the extensors known as posterior interosseous nerve entrapment syndrome
o steps to protect the PIN include
dissecting supinator off radius subperiostally
do not place retractors on posterior surface of radial neck
avoid excessive radial retraction of supinator
o injury
injury leads to a neuropraxia that takes 6-9 months to resolve
Henry’s approach to distal radius:
Incision along palpable FCR tendon sheath, curve at crease to prevent crossing perpendicular to flexion crease.
Dissect through FCR sheath, section fibres of tendon sheath in line with tendon, retract FCR tendon ulnarly and incise through the dorsal aspect of the FCR sheath. Expose pronator quadratus – caution of palmar cutaneous branch of the median nerve (arises 5cm proximal to wrist joint, ulnar to FCR).
Visualize the proximal extent of PQ and take down sharply with a knife – radial and distal borders in L-shape – caution branches of radial artery bleeding. Release brachioradialise
33.
A 60‐year‐old man has chronic shoulder pain and weakness. Radiographs show moderate glenohumeral arthritis and narrowing of the acromio‐humeral distance. He is scheduled to undergo either hemiarthroplasty or total shoulder arthroplasty.
Which of the following factors will most affect his postoperative function?
A. The integrity of the rotator cuff
B. The integrity of the coracoacromial ligament
C. The presence of glenoid wear
D. The presence of an inferior head osteophyte
E. The extent of AC joint arthritis
Answer: A. The integrity of the rotator cuff
TSA is indicated for cases of end-stage GH OA. It is preferred to hemiarthroplasty. It is contraindicated in cases with insufficient glenoid bone stock (glenoid wear to the level of the coracoid), rotator cuff arthropathy or irreparable cuff tears and deltoid dysfunction. It provides good pain relief and has good survival at 10 years (>90%).
TSA:
Replacement of humeral head and glenoid resurfacing: cemented all-polyethylene glenoid resurfacing is standard of care
Total shoulder arthroplasty unique from THA and TKA in that:
* Greater range of motion in the shoulder
* Success depends on proper functioning of the soft tissues
* Glenoid is less constrained: leads to greater sheer stresses and is more susceptible to mechanical loosening
Factors required for success of TSA
- Rotator cuff intact and functional: if rotator cuff is deficient and proximal migration of humerus is seen on x-rays (rotator cuff arthropathy) then glenoid resurfacing is contraindicated. If there is an irreparable rotator cuff deficiency then proceed with hemiarthroplasty or a reverse ball prosthesis. An isolated supraspinatus tear without retraction can proceed with TSA - incidence of full thickness rotator cuff tears in patients getting a TSA is 5% to 10% - if positive impingement signs on exam, order a pre-operative MRI.
- Glenoid bone stock and version
if glenoid is eroded down to coracoid process then glenoid resurfacing is contraindicated (Walch classification).
Outcomes
Pain relief more predictable than hemiarthroplasty, reliable range of motion, good survival at 10 years (93%), good longevity with cemented and press-fit humeral components, worse results for post-capsulorrhaphy arthropathy.
Indications
* Pain (anterior to posterior), especially at night, and inability to perform activities of daily living
* Glenoid chondral wear to bone: preferred over hemiarthroplasty for osteoarthritis and inflammatory arthritis
* Posterior humeral head subluxation
Contraindications
* Insufficient glenoid bone stock
* Rotator cuff arthropathy
* Deltoid dysfunction
* Irreparable rotator cuff (hemiarthroplasty or reverse total shoulder are preferable) as risk of loosening of the glenoid prosthesis is high (“rocking horse” phenomenon)
* Active infection
* Brachial plexus palsy
Glenoid loosening - most common cause of TSA failure (30% of primary OA revisions)
Risk factors: insufficient glenoid bone stock (posterior glenoid wear associated with glenoid loosening), rotator cuff deficiency
2.9% reoperation rate for loosening (28% with revision)
Vascular injury
Arcuate artery, branch off the anterior humeral circumflex artery, can be damaged during biceps tendon elevation
Humeral stem loosening: more common in RA and osteonecrosis. Rule out infection.
Subscapularis repair failure
Malposition of components
Improper soft tissue balancing: failure due to undiagnosed presence of rotator cuff tears
Iatrogenic rotator cuff injury can occur if humeral neck osteotomy is inferior to level of rotator cuff insertion. Overstuffing glenohumeral joint leading to attritional supraspinatus and subscapularis tears
Stiffness
Infection: may have normal aspiration results
culture. Infection rate 1-2% after primary TSA.
Arthroscopic tissue culture more sensitive (100% sensitive and specific) than fluoroscopically guided aspiration (17% sensitivity, 100% specific)
Propionibacterium acnes (P. acnes),
now referred to as Cutibacterium acnes (c. acnes) most common cause of indolent infections and implant failures - gram positive, facultative, aerotolerant, anaerobic rod that ferments lactose to propionic acid
has high bacterial burden around the shoulder forms biofilm.
P. acnes PJI more common in males
Use anaerobic culture bottles, keep for 10-14days (mean time to detection 6 days)
Neurologic injury: axillary nerve is most commonly injured, musculocutaneous nerve can be injured by retractor placement under conjoint tendon
Periprosthetic fracture: acceptable fragment alignment ≤ 20° flexion/extension, ≤ 30° varus/valgus, ≤ 20° rotation malalignment
Wright and Cofield system:
Type A fractures are proximal to the stem tip and are treated with ORIF;
Type B fractures are at the level of the stem tip and are treated with ORIF;
Type C fractures are distal to the stem tip and can be initially treated nonoperatively.
Steinmann et al. reviewed the treatment of periprosthetic humeral shaft fractures.
For intraoperative fractures, the authors recommended placement of a long stem prosthesis that bypasses the fracture site by at least 2 cortical diameters.
For postoperative type A and B fractures, treatment depends on whether the stem is loose or well fixed. Loose prostheses necessitate revision long stem component with supplementary fixation, whereas well-fixed stems require hybrid plate fixation. Type C fractures can be treated non-operatively, but in the presence of nonunion may require plate fixation with or without allograft struts.
- A shoulder hemiarthroplasty is a procedure in which the humeral articular surface is replaced with stemmed humeral component.
- The most common indication is glenohumeral arthritis when the glenoid bone stock is inadequate for a total shoulder arthroplasty.
- It is contraindicated in patients with coracoacromial ligament deficiency.
34.
The commonest tendon to be affected by tennis elbow is:
A. EDC
B. ECRL
C. ECRB
D. FCR
E. FDS
Answer: C. ECRB
Lateral Epicondylitis (also know as Tennis Elbow) is an overuse injury caused by eccentric overload at the origin of the common extensor tendon, leading to tendinosis and inflammation of the ECRB.
Diagnosis is made clinically with tenderness over the lateral epicondyle made worse with resisted wrist extension, gripping activites and decreased grip strength.
Treatment is primarily nonoperative with NSAIDs, activity modification and bracing (95% successful). Rarely, operative management is indicated for patients with persistent symptoms who fail nonoperative management.
Release and debridement of ECRB origin if failed prolonged non-operative management, with clear diagnosis. Incision over common extensor origin, lift ECRL off ECRB, excise degenerative tissue, decorticate epicondyle, repair capsule (if breached), side to side closure of tendon.
Complications: Iatrogenic LUCL injury – resection should not extend beyond equator of hradial head (may posterolateral rotatory insufficiency), Iatrogenic radial nerve injury, heterotopic ossification.
Most common cause for elbow symptoms in patients with elbow pain – tenodesis effect to optimize grip causes overuse of ECRB – precipitated by repetitive wrist extension and forearm pronation.
Usually begins as a micro-tear of the origin of ECRB, but may also inbolve microtears of ECRL and ECU.
Microscopy: angiofibroblastic hyperplasia, disorganized collagen, vascular hyperplasia.
Common extensor origin:
Lateral supracondylar ridge: ECRL
Lateral epicondyle: ERCB, ECU, ED, EDM, Anconeus(same attachment site as ECRB)
PIN enters the supinatpr just distal to the radial head – compression radial tunnel syndrome (associated in 5%)
35.
“Gagey’s sign” is elicited by stabilising the scapula with one hand and abducting the arm with the other.
Achieving abduction beyond 105 degrees is regarded as abnormal, and a sign of:
A. Inferior Laxity
B. Posterior Laxity
C. Superior Laxity
D. Anterior Laxity
E. A large Bony Lesion
Answer: A. Inferior Laxity
Gagey sign - hyperabduction test for assessment of the IGHL - range passive abduction >105 deg with 90 in the contralateral shoulder in 85% of patients with instability. An RPA of more than 105° is associated with lengthening and laxity of the inferior glenohumeral ligament.
Passive abduction occurs within the glenohumeral joint only, is controlled by the inferior glenohumeral ligament
Traumatic Anterior Shoulder Instability, also referred to as TUBS (Traumatic Unilateral dislocations with a Bankart lesion requiring Surgery), are traumatic shoulder injuries that generally occur as a result of an anterior force to the shoulder while its abducted and externally rotated and may lead to recurrent anterior shoulder instability.
Diagnosis is made clinically with the presence of positive anterior instability provocative tests and confirmed with MRI studies that may reveal labral and/or bony injuries of the glenoid and proximal humerus (Hill-Sachs lesion).
Treatment may be nonoperative or operative depending on the chronicity of symptoms, the presence of risk factors for recurrence, and the severity of labral and/or glenoid defects. In high-risk populations, surgery is often offered after a single dislocation event.
bankart lesion
is an avulsion of the anterior labrum and anterior band of the IGHL from the anterior inferior glenoid.
is present in 80-90% of patients with TUBS
bony bankart lesion
is a fracture of the anterior inferior glenoid
present in up to 49% of patients with recurrent dislocations
higher risk of failure of arthroscopic treatment if not addressed
defect >20-25% is considered “critical bone loss” and is biomechanically highly unstable
stability cannot be restored with soft tissue stabilization alone (unacceptable >2/3 failure rate)
requires bony procedure to restore bone loss (Latarjet-Bristow, other sources of autograft or allograft)
Humeral avulsion of the glenohumeral ligament (HAGL) occurs in patients slightly older than those with Bankart lesions
associated with a higher recurrence rate if not recognized and repaired
an indication for possible open surgical repair
glenoid labral articular defect (GLAD) is a sheared off portion of articular cartilage along with the labrum
presence is a risk factor for failure following arthroscopic stabilization procedures
anterior labral periosteal sleeve avulsion (ALPSA)
can cause torn labrum to heal medially along the medial glenoid neck
associated with higher failure rates following arthroscopic repair
common finding in patients with recurrent instability managed nonoperatively
97% of patients with recurrent instability have either a Bankart or ALPSA lesion
Hill-Sachs defect
is a chondral impaction injury in the posterosuperior humeral head secondary to contact with the glenoid rim.
is present in 80%-100% of traumatic dislocations and 25% of traumatic subluxations
Static restraints: bony anatomy, capsule, glenohumeral ligaments, labrum (labrum contributes 50% of additional glenoid depth)
Dynamic restraints: rotator cuff muscles & long head of biceps tendon
Anterior static shoulder stability is provided by:
1. Anterior band of IGHL (main restraint)
provides static restraint with arm in 90° of abduction and external rotation
2. MGHL - provides static restraint with arm in 45° of abduction and external rotation
3. SGHL - provides static restraint with arm at the side
36.
A patient slipped while cutting an object with a sharp knife and sustains a wound to the volar surface of her left index finger.
Exploration of the wound shows complete division of both FDS and FDP tendons over the proximal phalanx.
With respect to flexor tendon repair, which ONE of the following statements is TRUE?
A. Poor tendon handling does not lead to adhesion formation
B. The number of core suture strands affects the repair strength
C. The number of grasping loops affects the repair strength
D. The tendon sheath must always be repaired
E. The digit should be immobilised for three weeks
Answer: B. The number of core suture strands affects the repair strength
Flexor Tendon Injuries are traumatic injuries to the flexor digitorum superficialis and flexor digitorum profundus tendons that can be caused by laceration or trauma.
Diagnosis is made clinically by observing the resting posture of the hand to assess the digital cascade and the absence of the tenodesis effect. Treatment is usually direct end-to-end tendon repair.
Tendon healing - occurs via 2 pathways: (3 phases - inflammatory, fibroblastic, remodelling)
- Intrinsic: produced by tenocytes within the tendon
- Extrinsic: stimulated by surrounding synovial fluid and inflammatory cells. Implicated in the formation of scarring and adhesions
Camper chiasm: located at the level of the proximal phalanx where FDP splits FDS
Pulley system:
Digits 2-5:
- 5 annular pulleys (A1 to A5) which are thicker and stiffer than cruciate pulleys. A2 and A4 arise from the periosteum and are the most important pulleys to prevent flexor tendon bowstringing. A1, A3, and A5 arise from the volar plate
- 3 cruciate pulleys (C1 to C3) : collapsible and flexible - allows the annular pulleys to approximate each other during digital flexion
Whereas thumb contains 3 annular pulleys (A1, Av, A2) (A2 contributes least to arc of motion of thumb) and 1 interposed oblique pulley (most important pulley to prevent flexor tendon bowstringing (along with A1 pulley))
Tendons receive blood supply via 2 sources:
- Diffusion through synovial sheaths: occurs when flexor tendons are located within a sheath and is the more important source distal to the MCP joint
- Direct vascular perfusion nourishes flexor tendons located outside of synovial sheaths. Supplied by the vincular system, osseous bony insertions, reflected vessels from the tendon sheath, and longitudinal vessels from the palm
Verden Zones:
I: Distal to FDS insertion
II: FDS insertion to distal palmar crease/proximal A1 pulley
III: Palm (A1 pulley to distal aspect of carpal ligament)
IV: Carpal tunnel
V: Carpal tunnel to forearm
Indications for Flexor tendon repair:
> 75% laceration or ≥ 50-60% laceration with triggering (for these epitendinous suture at the laceration site is sufficient - no benefit of adding core suture)
Fundamentals of repair
- Easy placement of sutures in the tendon
- Secure suture knots
- Smooth juncture of the tendon ends
- Minimal gapping at the repair site
- Minimal interference with tendon vascularity
- Sufficient strength throughout healing to permit application of early motion stress to the tendon
Perform repair within three weeks of injury (2 weeks is ideal) - delayed treatment leads to difficulty due to tendon retraction
Approach: incisions should always cross flexion creases transversely or obliquely to avoid contractures (never longitudinal). Meticulous atraumatic tendon handling minimizes adhesions
Technique:
- Core sutures: # of suture strands that cross the repair site is more important than the number of grasping loops. Linear relationship between strength of repair and # of sutures crossing repair: 4-6 strands provide adequate strength for early active motion. High-caliber suture material increases strength and stiffness and decreases gap formation. Locking-loops decrease gap formation. Ideal suture purchase is 10mm from cut edge. Core sutures placed dorsally are stronger
- Circumferential epitendinous suture: improves tendon gliding by reducing the cross-sectional area. Improves strength of repair (adds 20% to tensile strength). Allows for less gap formation (first step in repair failure). Simple running suture is recommended. Produces less gliding resistance than other techniques
Sheath repair theoretically improves tendon nutrition through synovial pathway - (controversial) - clinical studies show no difference with or without sheath repair
Pulley management: historically believed to be critical to preserve A2 and A4 pulleys in digits and oblique pulley in thumb, although recent biomechanical studies have shown that 25% of A2 and 100% of A4 can be incised with little resulting functional deficit.
FDS repair: in zone 2 injuries, repair of one slip alone improves gliding compared to repair of both slips!
Repair failure
- Tendon repairs are weakest between postoperative day 6 and 12
- Repair usually fails at suture knots
- Repair site gaps > 3mm are associated with an increased risk of repair failure
Adhesion formation is increased risk with zone 2 injuries.
Index finger: Radial digital nerve is bigger (forms radial border of the hand) but paradoxically the ulnar digital artery is larger – survival advantage as less vulnerable!
37.
What structure is the primary stabilising actor of the glenohumeral joint in the human shoulder at 90° of abduction and maximal external rotation?
A. Anterior band of inferior glenohumeral ligament
B. Superior glenohumeral ligament
C. Coracohumeral ligament
D. Middle glenohumeral ligament
E. Posterior band of inferior glenohumeral ligament
Answer: A. Anterior band of inferior glenohumeral ligament
Anterior static shoulder stability is provided by:
1. Anterior band of IGHL (main restraint)
provides static restraint with arm in 90° of abduction and external rotation
2. MGHL - provides static restraint with arm in 45° of abduction and external rotation
3. SGHL - provides static restraint with arm at the side
Gagey sign - hyperabduction test for assessment of the IGHL - range passive abduction >105 deg with 90 in the contralateral shoulder in 85% of patients with instability. An RPA of more than 105° is associated with lengthening and laxity of the inferior glenohumeral ligament.
Passive abduction occurs within the glenohumeral joint only, is controlled by the inferior glenohumeral ligament
Traumatic Anterior Shoulder Instability, also referred to as TUBS (Traumatic Unilateral dislocations with a Bankart lesion requiring Surgery), are traumatic shoulder injuries that generally occur as a result of an anterior force to the shoulder while its abducted and externally rotated and may lead to recurrent anterior shoulder instability.
Diagnosis is made clinically with the presence of positive anterior instability provocative tests and confirmed with MRI studies that may reveal labral and/or bony injuries of the glenoid and proximal humerus (Hill-Sachs lesion).
Treatment may be nonoperative or operative depending on the chronicity of symptoms, the presence of risk factors for recurrence, and the severity of labral and/or glenoid defects. In high-risk populations, surgery is often offered after a single dislocation event.
bankart lesion
is an avulsion of the anterior labrum and anterior band of the IGHL from the anterior inferior glenoid.
is present in 80-90% of patients with TUBS
bony bankart lesion
is a fracture of the anterior inferior glenoid
present in up to 49% of patients with recurrent dislocations
higher risk of failure of arthroscopic treatment if not addressed
defect >20-25% is considered “critical bone loss” and is biomechanically highly unstable
stability cannot be restored with soft tissue stabilization alone (unacceptable >2/3 failure rate)
requires bony procedure to restore bone loss (Latarjet-Bristow, other sources of autograft or allograft)
Humeral avulsion of the glenohumeral ligament (HAGL) occurs in patients slightly older than those with Bankart lesions
associated with a higher recurrence rate if not recognized and repaired
an indication for possible open surgical repair
glenoid labral articular defect (GLAD) is a sheared off portion of articular cartilage along with the labrum
presence is a risk factor for failure following arthroscopic stabilization procedures
anterior labral periosteal sleeve avulsion (ALPSA)
can cause torn labrum to heal medially along the medial glenoid neck
associated with higher failure rates following arthroscopic repair
common finding in patients with recurrent instability managed nonoperatively
97% of patients with recurrent instability have either a Bankart or ALPSA lesion
Hill-Sachs defect
is a chondral impaction injury in the posterosuperior humeral head secondary to contact with the glenoid rim.
is present in 80%-100% of traumatic dislocations and 25% of traumatic subluxations
Static restraints: bony anatomy, capsule, glenohumeral ligaments, labrum (labrum contributes 50% of additional glenoid depth)
Dynamic restraints: rotator cuff muscles & long head of biceps tendon
38.
What is the most important factor affecting success of recovery following nerve repair?
A. Age of patient
B. Interval between injury and repair
C. Level of injury
D. Use of a fascicular repair
E. Condition of the nerve endings
Answer: A. Age of patient
Peripheral nerve injuries encompass a range of reversible and irreversible impairments determined by injury level, axonal disruption, and time to treatment.
Diagnosis can be made based on clinical examination and confirmed with EMG/NCS.
Treatment can involve observation, repair, tendon transfers or nerve grafting depending on the acuity, degree of injury, and mechanism of injury.
Prognosis:
- Pain is first modality to return – advancing Tinel sign is most reliable indicator of recovery – nerve repair or reconstruction is unpredictable after 6 months. Reinnervation by 18 months is the goal for muscle preservation.
- Variables: Younger age is the most important factor influencing success of nerve recovery, distal level of injury – second most important (more distal injury better chance of recovery), sharp transections and stretch injuries have better prognosis than crush or blast injuries)
- Negative: old age, proximal level injury, crush injury, repair delay
Major peripheral nerve injury is sustained in 2% of patients – increased with penetrating injuries and displaced fractures.
Stretching Injuries: 8% elongation diminishes nerve’s microcirculation, 15% elongation disrupts axons. E.g. Stingers = neuropraxia from brachial plexus stretch injury
Compression/Crush: fibres are deformed local ischaemia and increased vascular permeability. Endoneurial oedema leads to poor axonal transport and nerve dysfunction. Fibroblasts invade if compression persists – scar impairs fascicular gliding. Chronic compression leads to Schwann cell proliferation and apoptosis.
30mmHg paraesthesia (increased latencies)
60mmHg complete block of conduction
Laceration: sharp transection shave better prognosis than crush injuries. When the continuity of the nerve is disrupted the ends retract and nerve stops producing neurotransmitters as nerve starts producing proteins for axonal regeneration.
Regeneration: distal segment undergoes Wallerian degeneration (axoplasm and myelin are degraded by phagocytes), existing Schwann cells proliferate and line the endoneurial basement membrane. Proximal budding occurs after 1 month, and leads to sprouting axons that migrate at 1mm/day to the distal tube.
Functional recovery during regeneration (in order):
Sympathetic activity pain temperature touch proprioception motor function
Motor function is first lost and last to recover.
Schwann cells proliferate and trophic factors are upregulated to promote regeneration
pathoanatomy. Involvement of the axon, myelin, and supporting connective tissues influence regeneration potential - myelin disruption typically occurs before axon disruption. Axonal disruption leads to distal degeneration, requiring regeneration or repair to regain function
Neuronal connective tissue structure provides a framework for regeneration: endoneurium, perineurium, epineurium
Axillary nerve: shoulder dislocations
Radial nerve: distal third – Holsten Lewis fractures, prolonged compressed – Saturday night palsy, extension type supracondylar fractures.
Ulnar nerve: distal humerus ORIF, improper positing on OR table, flexion type supracondylar fracture
AIN: extension type supracondylar humerus fracture
Sciatic nerve: posterior hip dislocation
CPN: correction valgus alignment during TKA
Superficial peroneal nerve: percutaneous plating tibial fractures (holes 11-13)
Extrinsic blood supply: loose connective tissue surrounding nerve trunk
Intrinsic blood supply: plexus lies in epineurium, perineurium and endoneurium
Nerve structure:
- Epineural sheath: surrounds peripheral nerve
- Epineurium: surrounds group of fascicles to form a peripheral nerve – functions to cushion fascicles against external pressure
- Perineurium: connective tissue sheath covering individual fascicles, primary source of tensile strength and elasticity of a peripheral nerve, provides an extension of the blood-brain barrier.
- Fascicles: group of axons and surrounding endoneurium
- Endoneurium: loose fibrous tissue covering axons – participates in the formation of Schwann cell tube
- Myelin: made by Schwann cells, insulates axons to increase conduction velocity (conduction occurs at nodes of Ranvier)
- Neuron cell: cell body (metabolic centre making up <10% mass), axon (primary conducting vesicles), dendrites (thin branching processes that receive input from surrounding nerve cells).
Myelin is made by Schwann cells and acts to insulates axons to increase conduction velocity at nodes of Ranvier
Neuron cell - composed of cell body, axon and dendrites. Cell body - the metabolic center that makes up < 10% of cell mass, axon - primary conducting vehicle, dendrites - thin branching processes that receive input from surrounding nerve cells
Seddon Classification
- Neurapraxia: same as Sunderland 1st degree, “focal nerve compression” - nerve contusion or stretch leading to reversible conduction block without Wallerian degeneration. Usually caused by local ischemia - histopathology shows focal temporary demyelination of the axon (axon remains intact), endoneurium remains intact.
On electrophysiologic studies: nerve conduction velocity slowing or a complete conduction block, no fibrillation potentials
Recovery prognosis is excellent
- Axonotmesis : same as Sunderland 2nd-4th degree. Incomplete nerve injury more severe than neurapraxia. Axon and myelin sheath disruption leads to focal conduction block with Wallerian degeneration and variable degree of connective tissue disruption
Electrophysiologic studies show fibrillations and positive sharp waves on EMG
More unpredictable recovery. - Neurotmesis: encompasses Sunderland 5th degree - complete nerve division with disruption of endoneurium - all connective tissues disrupted, leads to a focal conduction block with Wallerian degeneration
Electrophysiologic studies show fibrillations and positive sharp waves on EMG
Generally no recovery unless surgical repair performed and neuroma formation at proximal nerve end may lead to chronic pain
Sunderland Classification
1st degree: same as Seddon’s neurapraxia (loss of myelin sheath)
2nd degree: included within Seddon’s axonotmesis - intact endoneurium, perineurium and epineurium
3rd degree: included within Seddon’s axonotmesis. The endoneurium injured with endoneurial scarring - intact perineurium and epineurium. Has most variable degree of recovery
4th degree: included within Seddon’s axonotmesis. The endoneurium and perineurium injured, with intact epineurium - nerve in continuity but at the level of injury there is complete scarring across the nerve. Generally unsatisfactory regeneration and may lead to neuroma-in-continuity
5th degree: same as Seddon’s neurotmesis - completely severed or transected nerve involving all layers - regeneration not possible without repair
39.
In closed reduction of posterior shoulder dislocation, which of the following is NOT true?
A. It should be forced into the external rotation.
B. Post reduction care should consist of a sling and a swathe if the shoulder is stable.
C. It usually requires general anaesthesia for reduction.
D. A shoulder spica may be used if the shoulder subluxed in a sling post‐reduction.
E. Intense physiotherapy is needed following a period of immobilisation post‐reduction.
Answer: A. It should be forced into the external rotation.
Posterior dislocations are more common following a seizure. The posteriorly dislocated shoulder is typically held in IR and most consistent finding is a mechanical block to ER caused by the anterior humeral head defect on the posterior aspect of the glenoid.
Acute reduction and immobilisation in ER for 4-6 weeks should be attempted for all acute traumatic posterior dislocations – most reduce spontaneously.
Technique: Immobilise in 10-20 deg ER with elbow at side, after 6 weeks advance to physio (rotator cuff strengthening and peri-scapular stabilisation) and activity modification (avoid activities that place arm in high-risk position).
According to the reference by Robinson et al, good functional outcomes are associated with early detection and treatment of isolated posterior dislocations that are associated with a small osseous defect and are stable following closed reduction.
Posterior shoulder instability and dislocations are less common than anterior shoulder instability and dislocations, but are much more commonly missed.
Diagnosis is made radiographically in the setting of acute dislocations. Chronic instability can be diagnosed with presence of positive posterior instability provocative tests and confirmed with MRI studies showing posterior labral pathology.
Treatment may be nonoperative or operative depending on chronicity of symptoms, recurrence of instability, and the severity of labrum and/or glenoid defects.
Risk factors for posterior dislocation include bony abnormality (glenoid retroversion or hypoplasia) and ligamentous laxity.
Mechanism:
Trauma - posterior dislocation - usually significant trauma.
Microtrauma - posterior instability - can lead to labral tear, incomplete labral avulsion or erosion of the posterior labrum which may lead to gradual stretching of the capsule and patulous posterior capsule - usually insidious onset and presentation.
Seizures and electric shock - tetanic muscles pull the humeral head out
Biomechanical forces: flexed, adducted and IR rotated arm = high risk position
Static restraint: labrum deepens glenoid by 50%
Primary stabilisers of posterior shoulder:
- Posterior band of IGHL: primary restraint in internal rotation
- Subscapularis: primary dynamic restraint in external rotation and against posterior subluxation
- Superior glenohumeral ligament and coracohumeral ligament: primary restraint to inferior translation of the adducted arm and to external rotation and primary static stabilizer to posterior subluxation with shoulder in flexion, adduction, and internal rotation.
Acute posterior dislocation -> limited external rotation, with the arm locked in an IR position. Pain on flexion, adduction and IR for posterior instability.
40.
When performing open reduction and internal fixation of a displaced fracture of the scapular neck, which of the following peripheral nerves is at the greatest risk of injury?
A. Axillary
B. Musculocutaneous
C. Upper subscapular
D. Radial
E. Dorsal scapular
Answer: A. Axillary
Axillary nerve is at risk inferior to the glenoid as it runs from anterior to posterior
Scapula neck fractures with associated ACJ separation or clavicle fracture = floating shoulder.
Conserv Mx if translation <1cm, angulation <40deg, glenopolar angle >20deg, no additional injury to shoulder suspensory complex (unstable nature bony/ligamentous ring).
Surg Mx scapula neck fractures has good shoulder function and high union rates, complication rate 15%.
Approaches: straight posterior overlying glenohumeral joint (less extensile than Judet) for isolated displaced neck and lateral scapular border fractures.
Judet approach: indicated when multiple scapula borders need to be accessed – incision courses along spine of scapula and angles down vertebral scapula border in ‘L’ shape. Internervous plane infraspinatus (suprascapular N.) and teres minor (axillary N.).
2.7mm or 3.5mm (locking) plate/recon plate to contour around scapula spine (thin scapula bone)
Nerves at risk with posterior/Judet approach: axillary N, suprascapular N, circumflex scapular V, postural humeral circumflex V.
Scapula Fractures are uncommon fractures to the shoulder girdle caused by high energy trauma and associated with pulmonary injury, head injury, and increased injury severity scores. >70% RTAs, other mechanisms: indirect trauma (FOOSH). Anterior glenohumeral dislocation anterior rim fracture, posterior dislocation posterior rim fracture.
Diagnosis can be made with plain radiographs and CT studies are helpful for fracture characterization and surgical planning.
Treatment is usually nonoperative with a sling. Surgical management is indicated for intra-articular fractures, displaced scapular body/neck fractures, open fractures, and those associated with glenohumeral instability.
Scapula body/spine = 45-50%
Glenoid neck = 25%, glenoid fossa/rim = 10% - often associated with impaction of humeral head into glenoid.
Acromion = 8%
Coracoid = 7%
Scapula functions to connect scapula to thorax, spine and upper extremity. Lateral triangle shape with 4 major processes.
- Scapula spine: osseous bridge separating supraspinatus and infraspinatus. Spinoglenoid notch represents possible site of compression for suprascapular nerve.
- Glenoid:articulating process on lateral scapula serving as a socket for glenohumeral joint, pear shaped and wider inferiorly from anterior to posterior, average 1-5 deg retroversion and 15deg upper tilt from scapula plane. Fibrocartilagenous labrum deepens glenoid fossa by 50% to increase stability.
- Acromion: articulates with clavicle to form ACJ – formed from 3 ossification centres; pre-acromion (tip), meso-acromion (mid), meta-acromion (base). Os acromiale = unfused secondary ossification centres (meso- and meta-acromion) – associated with impingement and rotator cuff symptoms.
- Coracoid process: has 2 secondary ossification centres that are open until around age 25 (don’t confuse with fracture) – angle of coracoid and tip of coracoid. Muscular attachments: Conjoint tendon (coracobrachialis, short head biceps), pec minor. Ligament attachments: Coracoclavicular (CC) ligaments - most anterior attachment is 25mm from tip of coracoid, and coracoacromial ligament.
41.
In which stage of adhesive capsulitis do the pathological findings include: full‐blown capillary proliferation with synovial hypertrophy and capsular adhesions?
A. Stage one: The “freezing” or painful stage
B. Stage two: The “frozen” or adhesive stage
C. Stage three: The “thawing” or recovery stage
D. Stage four: The “residual” stage
E. Stage five: The “stiffness” stage
Answer: B. Stage two: The “frozen” or adhesive stage
Arthroscopic Stagess:
Stage 1: patchy, fibrinous synovitis
Stge 2: capsular contraction and fibrinous adhesions
Stage 3: increasing contraction, synovitis resolving
Stage 4: severe contraction
Clinical Stages:
Freezing/painful: gradual onset of diffuse pain (6weeks – 9 months)
Frozen/stiff: decreased ROM affecting ADLs (4-9months or more)
Thawing: gradual return of motion (5-26months)
Adhesive capsulitis (also known as frozen shoulder) is a condition of the shoulder characterized by functional loss of both passive and active shoulder motion commonly associated with diabetes, thyroid disease, dupuytren’s disease, atherosclerotic disease and cervical disc disease. Most common in women 40-60 years.
Diagnosis is made clinically with marked symmetric loss of active and passive range of motion of the shoulder – slight ER rotation deficit most common finding – tethered end point to motion. Pain may be throughout motion arc or at terminal motion, depending on stage of disease.
XR to evaluate for OA, posterior dislocation or surgical hardware – may just show diffuse osteopenia. MRI is not necessary for diagnosis but can evaluate for other pathology. Loss of axillary recess indicates contracture of joint capsule.
Treatment is a prolonged course of aggressive physical therapy and medical management of underlying disease if present (i.e diabetes, thyroid disorder). Manipulation under anaesthesia or arthroscopic capsular release is indicated in patients with progressive loss of motion having failed a prolonged course of physical therapy.
May be primary, idiopathic form, post-traumatic (prox humerus fracture or immobilisation for other injury) or post-surgical (RC repair or axillary dissection for malignancy).
Inflammatory process causing fibroblastic proliferation of joint capsule leading to thickening, fibrosis and adherence of the capsule to itself and humerus. Fibroblasts/myofibroblasts with abundant type III collagen present leading to a mechanical block to motion.
Essential lesion involves the coracohumeral ligament and rotator interval capsule.
Rotator interval = triangular region between anterior border of supraspinaus and the superior border of subscapularis – contains SGHL and coracohumeral ligament.
Diabetes (Type 1 & 2) associated has worse outcome regardless of treatment, increases in older age, increased duration DM, autonomic neuropathy & history MI – may be first manifestation of DM
42.
Which of the following is true of a reverse polarity shoulder replacement?
A. It is used in the high demand patient with a strong rotator cuff
B. It is based on the idea of utilising the deltoid as the primary motor for the glenohumeral joint
C. It decreases the lever arm of the deltoid
D. It shifts the centre of rotation superiorly and laterally
E. Notching is not a common problem with this type of implant
Answer: B. It is based on the idea of utilising the deltoid as the primary motor for the glenohumeral joint
Reverse Shoulder Arthroplasty is a type of shoulder arthroplasty that uses a convex glenoid hemispheric ball and a concave humerus articulating cup to reconstruct the glenohumeral joint.
The center of rotation is moved inferiorly and medialized which allows the deltoid muscle to act on a longer fulcrum and have more mechanical advantage – compensating for the deficient RC muscles to provide shoulder abduction, but does not significantly help shoulder IR or ER. However, can combine with latissimus dorsi transfer to assist with ER.
Reverse Shoulder Arthroplasty is indicated for conditions such as rotator cuff tear arthropathy, pseudoparalysis due to cuff tear & arthritis, comminuted 4-part proximal humerus fractures in the elderly (poor bone healing potential), rheumatoid arthritis (if sufficient bone stock) and prior failed shoulder arthroplasty.
Patient characteristics: low functional demand, physiological age >70, sufficient glenoid bone stock, working deltoid (intact axillary nerve)
Contraindications: axillary nerve dysfunction, global deltoid deficiency (partial is relative CI), acromion deficiency, glenoid osteoporosis, active infection.
Scapular notching occurs in significant number of grammont style prosthesis – due to 155deg humeral component neck-shaft angle that effectively medializes the humeral component. There is a decreased incidence with lateralisation of the base-plate. Occurs as medial rim of the humeral cup impinges during adduction.
Risk factors: superior placement & tilt of glenoid component, medialisation of COR, high BMI.
Dislocation is the commonest cause of early failure – risk factors: irrepairable subscap (strongest risk), proximal humeral bone loss, failed prior arthroplasty, proximal humeral non-union, fixed pre-op glenohumeral dislocation. Position of dislocation is most commonly extension, IR and adduction.
Glenoid loosening is commonest mechanism of failure
Deep infection 1-2% risk – most common organism c.acnes and staphylococci.
43.
Which of the following clinical findings is LEAST likely to be associated with a pre‐ganglionic complete brachial plexus injury?
A. Bruising in the anterior triangle of the neck
B. Pain in an insensate hand
C. Loss of sensation above the clavicle
D. Ipsilateral Horner’s syndrome
E. Loss of muscle function of branches direct from the roots of the brachial plexus
Answer: A. Bruising in the anterior triangle of the neck
The trunks are located in the posterior triangle of the neck, enclosed by the posterior border of the sternocleidomastoid muscle, anterior border of the upper trapezius, and clavicle.
Brachial plexus injuries (BPIs) can involve any degree of injury at any level of the plexus and range from obstetric injuries to traumatic avulsions.
Diagnosis requires focused physical examination with EMG/NCS and MRI studies used for confirmation as needed.
Treatment can be conservative versus operative depending on the age of patient, chronicity of injury, degree of injury and nerve root involvement.
Supraclavicular injuries:
- Complete involvement all roots: 75-80% traumatic BPIs
- C5 and C6 upper trunk (Erb’s palsy): 20-25% traumatic BPIs
- C8, T1 or lower (Klumpke palsy): 0.6-3% traumatic BPIs
Usually high speed (mostly motorcycle) vehicle accidents
- Caudally forced shoulder predominantly upper brachial pleux
- Forced arm abduction (e.g. grabbing while falling) predominantly lower roots
Pre-ganglionic: avulsion injury to dorsal root ganglion – involves CNS which does NOT regenerate – little potential of motor recovery (poor prognosis). EMG may show loss of innervation to cervical paraspinals.
Lesions suggesting pre-ganglionic injury:
- Horner syndrome (sympathetic chain disruption) – usually shows up 3 days post injury.
- Winged scapula medially (inferior border goes medial - loss of serratus anterior – long thoracic nerve), N.B. loss of rhomboids (dorsal scapula nerve) leads to lateral winging – superior medial border drops downwards and protrudes laterally and posteriorly. Important to test both – if both working then likely lesion C5 is post-ganglionic.
- Motor deficit (flail arm) – both pre- and post-ganglionic lesions can present with flail arm
- Sensory absent – severe pain in anaesthetised limb correlates with root avulsion.
- Absence of Tinel sign or tenderness to percussion in the neck
- Normal histamine test (C8-T1 sympathetic ganglion) – intact triple response (redness, wheal, flare)
- Elevated hemi-diaphragm (phrenic nerve)
- Rhomboid paralysis (dorsal scapular nerve)
- Normal sensory nerve action potentials (SNAP)
Post-ganglionic: involve PNS, capable of regeneration, therefore a better prognosis.
Presents with motor deficit (flail arm) and sensory deficits.
EMG shows maintained innervation to cervical paraspinals.
Abnormal histamine test: only redness and wheal, but no flare.
44.
In surgical management of shoulder instability; which of the following statements is FALSE
A. The Bankart procedure involves reattachment of the labrum and the Inferior Glenohumeral Ligament Complex to the glenoid.
B. The Putti‐Platt procedure results in decreased internal rotation.
C. The Laterjet procedure involves transfer of the coracoid tip to the anterior glenoid bone defect.
D. Capsular shift involves tightening the inferior capsule by shifting superiorly, or “pants over vest”.
E. Capsular shift is the gold standard surgical procedure for multi‐ directional instability
Answer: B. The Putti‐Platt procedure results in decreased internal rotation.
Traumatic Anterior Shoulder Instability, also referred to as TUBS (Traumatic Unilateral dislocations with a Bankart lesion requiring Surgery), are traumatic shoulder injuries that generally occur as a result of an anterior force to the shoulder while its abducted and externally rotated and may lead to recurrent anterior shoulder instability.
Diagnosis is made clinically with the presence of positive anterior instability provocative tests and confirmed with MRI studies that may reveal labral and/or bony injuries of the glenoid and proximal humerus (Hill-Sachs lesion).
Treatment may be nonoperative or operative depending on the chronicity of symptoms, the presence of risk factors for recurrence, and the severity of labral and/or glenoid defects. In high-risk populations, surgery is often offered after a single dislocation event.
Bankart lesion is an avulsion of the anterior labrum, and anterior band of IGHL from the anterior inferior glenoid – present in 80-90% of patients with TUBS.
Bony bankart lesion is a fracture of the anterior inferior glenoid present in up to 49% of patients with recurrent dislocations. Higher risk of failure of arthroscopic treatment if not addressed
Defect >20-25% is considered “critical bone loss” and is biomechanically highly unstable
Stability cannot be restored with soft tissue stabilization alone (unacceptable >2/3 failure rate)
Requires bony procedure to restore bone loss (Latarjet-Bristow, other sources of autograft or allograft)
Humeral avulsion of the glenohumeral ligament (HAGL) occurs in patients slightly older than those with Bankart lesions - associated with a higher recurrence rate if not recognized and repaired. An indication for possible open surgical repair
Glenoid labral articular defect (GLAD) is a sheared off portion of articular cartilage along with the labrum - presence is a risk factor for failure following arthroscopic stabilization procedures
Anterior labral periosteal sleeve avulsion (ALPSA) can cause torn labrum to heal medially along the medial glenoid neck. Associated with higher failure rates following arthroscopic repair.
Common finding in patients with recurrent instability managed nonoperatively - 97% of patients with recurrent instability have either a Bankart or ALPSA lesion
Hill-Sachs defect is a chondral impaction injury in the posterosuperior humeral head secondary to contact with the glenoid rim. Present in 80%-100% of traumatic dislocations and 25% of traumatic subluxations. (Hill-Sachs defect is ‘off-track’ and will ‘engage’ on the glenoid if the size of the Hill-Sachs defect > glenoid articular track, whereas if Hill-Sachs defect is ‘on-track’ and will not engage if the size of the hill-Sachs defect < glenoid articular track) – goal of treatment is to convert an off-track lesion into an on-track one.
Static restraints: bony anatomy, capsule, glenohumeral ligaments, labrum (labrum contributes 50% of additional glenoid depth)
Dynamic restraints: rotator cuff muscles & long head of biceps tendon
Anterior static shoulder stability is provided by:
1. Anterior band of IGHL (main restraint)
provides static restraint with arm in 90° of abduction and external rotation
2. MGHL - provides static restraint with arm in 45° of abduction and external rotation
3. SGHL - provides static restraint with arm at the side
Arthroscopic Bankart repair +/- capsular plication:
Relative indications: 1st time traumatic dislocation with Bankart lesion on MRI in athlete <25years, high demand athletes, recurrent dislocation/subluxation, <20-25% bone loss.
Remplissage augmentation with arthroscopic Bankart may be considered if Hills-Sach ‘Off-track”
At least 3 (preferably 4) anchor points should be used – labrum must be fully mobilised prior to repair. Too many anchor points pose risk fracture through the anterior holes – postage stamp fracture.
Equally efficacious to open repair with the advantage of less pain and greater motion preservation.
Open for revision stabilisation following failed arthroscopic Bankart repair without glenoid bone loss >20% - deltopectoral approach – can fix bony Bankart with screws or suture in a linear or bridge technique.
Laterjet (coracoid transfer)
Indications: chronic bony deficiencies >20-25% glenoid deficiency – inverted pear deformity to glenoid. When extensive bone loss has occurred, excessive stress is transferred to labrum and attenuated anterior soft tissues, increasing the risk of failure of labral repair alone.
Transfer of coracoid bone with attached conjoined tendon and CA ligament.
Laterjet triple effect: bony (increases glenoid track), sling (conjoined tendon on top of subscapularis), capsule reconstruction (CA ligament). Deltopectoral approach – subscap is split. Good to excellent results in >90% patients.
Putti-platt is now a historical procedure as led to over-constraint & arthrosis – goal is to tighten the subscapularis by lateral advancement of subscapularis and medial advancement of the shoulder capsule.
Typical post-op presentation of pain, stiffness (from GHJ OA) – especially lack of ER, and significant posterior glenoid wear and retroversion.
Multidirectional shoulder instability (MDI) is a condition characterized by generalized instability of the shoulder in at least 2 planes of motion (anterior, posterior, or inferior) due to capsular redundancy.
Diagnosis is made clinically with presence of increased anterior and posterior humeral translation, a sulcus sign, and overall increased external rotation.
Treatment is a trial of prolonged physical therapy focusing on dynamic stabilization and periscapular muscle training. Arthroscopic stabilization with capsular shift is indicated for patients with persistent instability who fail an extensive course of physical therapy.
Capsular shift: must address capsule +/- rotator interval
- Inferior capsular shifted superiorly, plication of redundant fashion and closure of rotator interval (produces the most significant decrease in ROM in ER with the arm at the side).
45.
This 12-year-old boy presents 6 weeks following a fall from a bicycle. On examination his knee is pain free and his range of motion lacks 40 degrees of extension with a mechanical block. He has a positive Lachman test. X-rays and MRI scans have been undertaken.
What is the most appropriate management for this patient?
(IMAGE)
A. Physiotherapy referral
B. Anterior cruciate ligament reconstruction
C. Arthroscopic/ open reduction (+/- osteotomy) and fixation
D. Posterior cruciate ligament reconstruction
E. Bucket handle meniscal tear reduction and repair
Answer: C. Arthroscopic/ open reduction (+/- osteotomy) and fixation
A Tibial Eminence Fracture, also known as a tibial spine fracture, is an intra-articular fracture of the bony attachment of the ACL on the tibia that is most commonly seen in children from age 8 to 14 years during athletic activity.
Diagnosis can be confirmed with radiographs of the knee. MRI studies can be helpful for determining associated ligamentous/meniscal damage (15-37% of fractures associated intra-articular pathology).
Treatment is closed reduction and casting or open reduction and fixation depending on the degree of displacement and success of closed reduction.
Tibial eminence: non-articular portion of tibia between the medial and lateral tibial plateau, consists of 2 spines with the ACL attaching to the medial spine. ACL insertion is 9mm posterior to the intermeniscal ligament and adjacent to anterior horns of meniscus. PCL does not attach to tibial spines.
In the paediatric population, the inter-condylar eminence is incompletely ossified and therefore more prone to failure than ligamentous structures – failure occurs through deep cancellous bone. Fractures are usually confined to intercondylar eminence, but may propagate to tibial plateau (most commonly medial).
Modified Meyers & McKeever Classification
Type I: Non-displaced <3mm
Type II: Minimally displaced with intact posterior hinge
Type III: completely displaced
Type III+: type III fracture with rotation
Type IV: completely displaced, rotated, comminuted
Ex: severe swelling and pain in knee, inability to WB – immediate effusion due to haemarthrosis – knee usually on flexed position. ROM limited due to pain, but may indicate meniscal pathology/displaced/entrapped fracture fragment/positive anterior draw test
Closed reduction in non-displaced Type I and reducible type II fractures then immobilise in cast in extension for 3-4 weeks.
All-arthroscopic/open fixation is indicated for Type II or III fractures that cannot be reduced (entrapped medial meniscus/intermeniscal ligament or pull of lateral meniscus attachment) or block to extension.
Fixation with sutures (e.g. through base of ACL) – minimal damage to physis but technically demanding or screws – less technically demanding but requires larger osteochondral fragment, hardware irritation, not possible for small, comminuted fragments, improper screw placement can cause impingement), iatrogenic comminution, physeal damage.
46.
Concerning the safe use of intraoperative tourniquets, which one of the following is TRUE?
A. Patients <16 years should have a tourniquet pressure of limb occlusion pressure plus 50 mm Hg or systolic blood pressure plus 50-100 mmHg.
B. Torniquets should be more than a quarter of the limb diameter.
C. Patients >16 years should have a tourniquet pressure of systolic blood pressure plus 50 -100 mm Hg for the lower limb
D. The ischaemic tourniquet time should ideally be less than 150 minutes and only extended beyond this after a clinical assessment of the relative risks and benefits, by the operating surgeon
E. The use of tourniquets is at the surgeon’s preference
Answer: A. Patients <16 years should have a tourniquet pressure of limb occlusion pressure plus 50 mm Hg or systolic blood pressure plus 50-100mmHg.
BOAST – the safe use of intra-operative tourniquets
Local tissue damage is a significant potential consequence of tourniquet use, particularly in vulnerable patients. All users should be aware of strategies for the prevention, diagnosis and management of tourniquet related injuries and that their early appreciation is imperative. This may be particularly challenging in patients undergoing regional anaesthesia and in patients unable to communicate adequately.
Inclusions:
All patients undergoing a procedure that involves application of a tourniquet.
Exclusions:
Trauma patients with pre-hospital tourniquet application for exsanguinating vascular injury.
Standards
1. Tourniquets should only be used when clinically justified.
2. Details of the type of tourniquet should be recorded.
a. Only tourniquets approved by regulatory bodies should be used.
b. Tourniquet width should be more than half the limb diameter or contoured for patients with conical limbs.
c. Finger or toe tourniquets should be highly visible or applied using instruments included in the surgical instrument count so that they cannot be inadvertently retained.
3. The following details should be recorded in the operative record:
a. The condition of the tourniquet site prior to and at the end of the procedure.
b. The method of isolation used to exclude skin preparation fluids from seeping under the tourniquet.
c. The method of exsanguination:
i. Compressive exsanguination should not be used in the presence of infection, history of malignancy or risk of DVT.
d. The pressure and duration of tourniquet use:
i. A limb tourniquet with a timer alarm should be used.
ii. If a pneumatic tourniquet is utilised, a pressure gauge must be used.
iii. Tourniquets should be applied over a thin, even layer of padding.
iv. Patients <16 years should have a tourniquet pressure of limb occlusion pressure plus 50 mm Hgi or systolic blood pressure plus 50-100 mmHg.
v. Patients >16 years should have a tourniquet pressure of systolic blood pressure plus 70-130 mmHg for the lower limb and 50 -100 mm Hg for the upper limb.ii,iii
vi. The ischaemic tourniquet time should ideally be less than 120iv minutes and only extended beyond this after a clinical assessment of the relative risks and benefits, by the operating surgeon. Audible reminders must be given to the operating surgeon every 10 minutes beyond 120 minutes, and tourniquet use beyond 150 minutes is rarely justified.
4. If a tourniquet related burn is suspected in the operating theatre, the following steps must be taken at the conclusion of the procedure:
a. Detailed documentation of the site and dimension of the injury.
b. Documentation of skin preparation fluid including duration of contact.
c. Digital photography, uploaded to the patient record.
d. Discussion with a plastic surgical and/or tissue viability team.
5. If a tourniquet related burn is confirmed, an ongoing management plan should be documented. This must include shared decision making with a plastic surgical and/or tissue viability team.
6. If tourniquet related ischaemia and/or nerve damage are suspected refer to the condition specific BOAST.
47.
Which of the following factors does NOT enhance the stability of circular external fixators.
A. Utilising smooth wires with narrow crossing angles
B. Use of olive wires
C. Additional use of half pins
D. Decreasing the distance from the fracture to the ring fixation elements
E. Increased number of rings in the construct
Answer: A. Utilising smooth wires with narrow crossing angles
Factors that increase stability of conventional external fixators:
- Larger diameter pins (most important)
- Contact ends of fracture
- Additional pins
- Decreased bone to rod distance
- Pins in different planes
- Increasing size or stacking rods
- Rods in different planes
- Increased spacing between pins
Factors that increase stability of circular external fixators:
- Larger diameter wires
- Decreased ring diameter
- Olive wires
- Extra wires
- Wires cross perpendicular to each other
- Increased wire tension (tensioned wires produce more axial compression with less interfragmentary shear than half pins)
- Placement of 2 central rings close to fracture
- Increased number of rings
48.
Regarding the Kocher‐Langenbeck approach to the acetabulum, which one of the following is TRUE?
A. During the approach, the hip should be held in extension with the knee flexed
B. The superior gluteal artery is rarely at risk as it emerges beneath the piriformis muscle
C. The Kocher‐ Langenbeck approach is a nonextensile approach to the anterior acetabular column
D. Detachment of the short external rotators of the hip is not required
E. In order to reduce tension on the sciatic nerve during the approach, the leg should be adducted and internally rotated
Answer: A. During the approach, the hip should be held in extension with the knee flexed
49.
In the multiply injured patient which of the following statements is TRUE?
A. Blood loss is the most serious complication of chest trauma resulting in airway compromise.
B. Moderate head injuries includes patients with a GCS of 7 and above.
C. Neurogenic shock results from impairment of the descending sympathetic pathways in the spinal cord.
D. Central cord syndrome is characterised by paraplegia and a dissociated sensory loss of pain and temperature sensation.
E. Scapular fracture suggests relatively mild trauma.
Answer: C. Neurogenic shock results from impairment of the descending sympathetic pathways in the spinal cord.
Neurogenic shock is characterized by hypotension & relative bradycardia in patient with an acute spinal cord injury - potentially fatal
Mechanism: circulatory collapse from loss of sympathetic tone. The disruption of autonomic pathway within the spinal cord leads to lack of sympathetic tone, decreased systemic vascular resistance, pooling of blood in extremities and hypotension
Treatment: Swan-Ganz monitoring for careful fluid management and pressors to treat hypotension
The most serious complication of chest trauma leading to airway compromise is the airway compromise!
GCS score of 8 or less Moderate Head Injury—-GCS score of 9 to 12 Mild Head Injury—-GCS score of 13 to 15.
Central cord syndrome is the most common incomplete cord injury – often in the elderly with minor extension injury mechanisms – due to anterior osteophytes and posterior infolded ligamentum flavum.
Spinal cord compression and central cord oedema with selective destruction of lateral corticospinal white tract matter- preferentially affecting hand and upper extremities (as these are located centrally in corticospinal tract.
Presentation: weakness with hand dexterity most affected, hyperpathia (burning in distal upper extremity), motor deficit worse in UL than LL, hands more pronounced than arms. Sacral sparing. UL has LMN signs (clumsy), LL has UMN signs (spastic).
Good prognosis, although full functional recovery is rare – usually regain bladder control, ambulatory but upper extremity and hand recovery is unpredictable – often have permanent clumsy hands.
Recovery in a typical pattern: first: lower extremity bowel and bladder function proximal upper extremity hand function last
50.
The AO principles of fracture care include all of the following EXCEPT
A. Anatomic reduction of the fracture fragments
B. Stable internal fixation
C. Non weight‐bearing until radiological fracture union
D. Preservation of blood supply
E. Early active mobilisation
Answer: C. Non weight‐bearing until radiological fracture union
Original AO principles
* Restoration of anatomy.
* Stable fracture fixation.
* Preservation of blood supply.
* Early mobilization of the limb and patient.
51.
A 50‐year old patient involved in a road traffic accident presents with class II shock.
Which of the following sets of signs would they have?
A. Pulse 80, BP 130/70, Respiratory rate 17, Normal mental status
B. Pulse 110, BP 110/95, Respiratory rate 24, Anxious
C. Pulse 125, BP 90/70, Respiratory rate 32, Confused
D. Pulse 145, BP 90/70, Respiratory rate 40, Unconscious
E. Pulse 90, BP 120/80, Respiratory rate 30, Confused
Answer: B. Pulse 110, BP 110/95, Respiratory rate 24, Anxious
Classification of haemorrhagic shock
o Type I
o Volume of blood loss (ml): <750
o Percentage blood loss (%): <15
o Heart rate (beats/min): <100
o Blood pressure: normal
o Pulse pressure: normal/increased
o Respiratory rate (breaths/min): 14-20
o Urine output (ml/hour): >30
o Mental state: slightly anxious
o Type II
o Volume of blood loss (ml): 750-1500
o Percentage blood loss (%): 15-30
o Heart rate (beats/min): 100-120
o Blood pressure: normal
o Pulse pressure: decreased
o Respiratory rate (breaths/min): 20-30
o Urine output (ml/hour): 20-30
o Mental state: mildly anxious
o Type III
o Volume of blood loss (ml): 1500-2000
o Percentage blood loss (%): 30-40
o Heart rate (beats/min): 120-140
o Blood pressure: decreased
o Pulse pressure: decreased
o Respiratory rate (breaths/min): 30-40
o Urine output (ml/hour): 5-15
o Mental state: anxious, confused
o Type IV
o Volume of blood loss (ml): >2000
o Percentage blood loss (%): >40
o Heart rate (beats/min): >140
o Blood pressure: decreased
o Pulse pressure: decreased
o Respiratory rate (breaths/min): >35
o Urine output (ml/hour): negligible
o Mental state: confused, lethargic
52.
Fixation of a Bennett fracture/dislocation must overcome the deforming forces of which of the following muscles:
A. Abductor pollicis longus & first dorsal interosseus
B. Abductor pollicis brevis & flexor pollicis longus
C. Flexor pollicis brevis & adductor pollicis
D. Abductor pollicis longus & adductor pollicis
E. Extensor pollicis longus & first dorsal interosseus
Answer: D. Abductor pollicis longus & adductor pollicis
Base of Thumb metacarpal fractures can be extra-articular fractures, Bennett fractures (partial intra-articular – palmar ulnar fragment), or Rolando fractures (Y or T shaped complete intra-articular). Mechanism is usually axial force applied to the thumb in pronation. Imperfect reductions lead to increased joint contact pressures and therefore predisposition to early arthritis. Excessive angulation may lead to MCPJ hyperextension deformity.
CMCJ = saddle shaped joint, composed of trapezium and base of the 1st metacarpal of the thumb – movements are flexion-extension and abduction-adduction.
3 Muscles provide the deforming forces at the base of the thumb:
- ABDuctor pollicis longus – PIN proximal, dorsal and radial force on shaft fragment
- Extensor pollicis longus – PIN proximal, dorsal and radial force on shaft fragment
- Adductor Pollicis – ulnar nerve supination and adduction force on the shaft fragment
Ligaments:
- Volar beak ligament: spans tuberosity of the trapezium to volar edge of 1st metacarpal. Keeps trapezium connected to the volar-ulnar based fragment.
- Dorsoradial ligament: spans dorsoradial tubercle of trapezium to dorsal base of 1st metacarpal.
53.
Which one of the following statements regarding the surgical treatment of patella fractures is TRUE?
A. Partial patellectomy is recommended for stellate patella fractures
B. Circumferential cerclage wiring is recommended for comminuted fractures
C. Longitudinal cannulated screws combined with tension band wires produces no biomechanical benefit compared with either treatment alone.
D. Does not allow early knee mobilisation
E. Total patellectomy reduces quadriceps strength by 20%
Answer: B. Circumferential cerclage wiring is recommended for comminuted fractures
Patella Fractures are traumatic knee injuries caused by direct trauma or rapid contracture of the quadriceps with a flexed knee that can lead to loss of the extensor mechanism.
Diagnosis can be made clinically with the inability to perform a straight leg raise and confirmed with radiographs of the knee.
Treatment is either immobilization or surgical fixation depending on fracture displacement and integrity of the extensor mechanism.
Mechanism Injury:
- Direct impact: fall, dashboard injury, high energy mechanism comminuted fracture pattern with chondral damage. Retinaculum may be intact.
- Indirect eccentric contraction: occurs following rapid knee flexion against a contracted quadriceps muscle failure in tension, often results in transverse fracture or inferior pole avulsion, retinacular injury is typical.
- Patella sleeve fracture: paediatric population
Patella is largest sesamoid bone in the body. Superior ¾ of posterior surface is covered by articular cartilage – thickest in the body (up to 1cm), inferior ¼ devoid of cartilage. Posterior articular surface is composed of 2 facets – medial and lateral (larger) separated into smaller facets and divided by vertical ridge.
Bipartite patella occurs in approx. 2-3% of population – usually superolateral.
MPFL: originates between medial epicondyle and adductor tubercle on femur and attaches to approximately upper 2/3 of medial patella – acts as primary ligamentous restraint to lateral patella translation. Most effective from 0-30deg flexion before patella engages trochlear groove.
Quadriceps tendon & fascia lata attach to anterosuperior margin of patella – tendon composed of 3 layers: superficial (rectus femoris), middle (vastus medialis & lateralis tendons), deep (vastus intermedius).
Retinaculum is formed by fascia lata, vastus medialis and vastus lateralis.
Patella offers biomechanical advantage to extensor mechanism by 30-50% by displacing it anteriorly away from the centre of rotation. During knee flexion, patella experiences tension from quadriceps and patella tendon, and compressive loads across posterior patella.
Surgery aims to preserve patella whenever possible. Partial patellectomy is only recommended when ORIF is not possible – aiming to remove the least bone possible, and the patella tendon advanced into defect on anterior surface of patella. Decrease in strength of extensor mechanism related to size of fragment removed.
TBW: converts tensile forces generated by quadriceps complex at anterior surface into compressive forces at the articular surface.
- K-wires & 18-gauge stainless steel wire: difficult to manipulate and high re-op rate (painful hardware, wire migration)
- K-wires & suture: 75% tensile strength of above, but performs similar clinically with lower rates hardware removal
- Tension band with longitudinal 4mm cannulated screws is biomechanically stronger than both.
Plate/screw construct: biomechanically superior to TBW construct.
- Mini-frag plates: useful in simple/comminuted fractures and helpful in osteoporotic bone.
Cerclage wiring: used alone or to augment additional fixation such as interfragmentary lag screws of TBW – useful in comminuted fractures.
54.
During percutaneous stabilisation of a distal radius fracture, placing a ‘K’ (Kirschner) wire >5mm ulnar to Lister’s tubercle may cause injury to which structure:
A. Extensor Pollicis Longus
B. Extensor Carpi Ulnaris
C. Extensor Digitorum Communis
D. Extensor Carpi Radialis Longus
E. Extensor Digiti Minimi
Answer: C. Extensor Digitorum Communis
Extensor compartments:
1: EPB, APL – de quervains
2:ECRB, ECRL – intersection syndrome
3: EPL – Drummer’s wrist, traumatic rupture (DR#)
4: Extensor indicis, EDigitorum – Extensor tenosynovitis
5: EDM – Vaighan-Jackson syndrome
6: ECU – snapping ECU